Hematology

You might also like

Download as docx, pdf, or txt
Download as docx, pdf, or txt
You are on page 1of 189

1.

1.

A patient is found to have an aggressive B-lymphocyte neoplasm after presenting to


his primary care physician with a rapidly growing splenic mass. The results of a
lymph node biopsy are show. Based on the result and the patient`s history of several
years of fatigue and weight loss, the patient`s current lymphoma is believed to have
transformed from a more benign lymphoma, which went undiagnosed . An analysis
of the lymphoid cell surface markers reveals a unique cell surface marker , CD5,
which is shared by these two lymphomas.

What is the diagnosis of the patient`s current lymphoma?

A.Burkitt lymphoma

B. *Diffuse large B-cell lymphoma

C. Follicular lymphoma

D. Mantle cell lymphoma

E. Peripheral T-lymphocyte lymphoma

1.

У пацієнта знайдено агресивне B-лімфоцитарне новоутворення і, крім того,


сімейний лікар виявив швидке збільшення селезіни. Отримано результати
біопсії лімфатичних вузлів. На підставі результатів і історії хвороби (декількох
років підвищеної втоми і втрати ваги)виявилось, що нинішня лімфома хворого,
як вважають, перетворилася з більш доброякісної лімфоми, яка не
діагностувалась, у злоякісну . Аналіз маркерів поверхні лімфоїдних клітин
виявляє унікальний клітинний маркер CD5, який поділяє ці дві лімфоми.
Який тип поточної лімфоми пацієнта?

А. Лімфома Беркіта

B. *Дифузна В-великоклітинна лімфома

C. Фолікулярна лімфома

D. Лімфома мантійних клітин

E. Периферична Т-лімфоцитарна лімфома


2.

2.

A 28-year-old African-American man presents with jaundice and dark urine 2 days
after starting primaquine for his upcoming trip to sub- Saharan Africa.He denies any
comorbidity and regular medication use.He recalls an older brother having a similar
episodè prior to his trip to the same region, but not his younges sister.On
examination, his vital signs are stable and he is afebrile.There is mild conjunctival
pallor and scleral icterus.There is no palpable lymphadenopathy or
organomegaly.Laboratory tests show:
WBC count: 8400/mm³

Hemoglobin: 12 g/dl

Hematocrit : 36 %

Platelet count: 289,000/mm³

Production of which of the following molecules is decreased in this condition?

A. Free radicals

B. Galactose-1-phosphate

C. Pyruvate

D.*The reduce form of nicotinamide adenine dinucleotide phosphate

E. Tyrosine

2. 28-річний афро-американець скаржиться на жовтяницю і темну сечю через 2


дні після початку прийому Примакіна для своєї майбутньої поїздки в Африку
на південь від Сахари. Він заперечує будь-яку супутню патологію і регулярне
використання ліків . Він згадав, що його старший брат мав схожі епізоди до
своєї поїздки в той же регіон, але не його молодша сестра .На огляді виявлено,
що його життєво важливі ознаки стабільні і гарячка відсутня .Є легка блідість
кон`юнктиви та іктеричність склер. При пальпації органи не збільшені,
патології лімфатичних вузлів не виявлено. Лабораторні тести показують:

Кількість лейкоцитів: 8400 / мм³

Гемоглобін: 12 г / дл

Гематокрит: 36%

Кількість тромбоцитів: 289,000 / мм³

Продукція якої з наступних молекул знижується при цій патології?

А. Вільні радикали

B. Галактоза-1-фосфат

С. Піруват

D. *Нікотинамідідендінуклеотидфосфату(НАДФ)

E. Тирозин
3.
3.

An 86-years-old man presents to his physician following 4 months of increasing


fatigue, a 6.8-kg (15-lb) unintentional weight loss, and an uncomfortable sense of
fullness in the left upper quadrant. Medical history is significant for hypertension and
dyslipidemia,which are both well-controlled on medication. He denies any history of
smoking. On examination, the vital signs are stable and afebrile. There is conjunctival
pallor, but no lymphadenopathy is present.A left upper quadrant mass is felt on
palpation. A complete blood cell count shows:

WBC count: 14,000/mm³

Hemoglobin: 9 g/dl

Hematocrit : 30 %

Platelet count: 470,000/mm³

Several attempts at bone marrow aspiration were unsuccessful. A peripheral blood


smear is shown.

Which of the following processes is associated with this patient's condition?

A. Aplastic anemia

B. Asplenia

C. Hemoglobin clumping

D. Intravascular hemolysis

E. *Myelofibrosis

3. 86-річний чоловік звернувся до лікаря після 4 місяців посиленої


втомлюваності, втрати ваги на 6,8 кг, і неприємне відчуття стискання у лівому
верхньому квадранті живота .В історія хвороби зазначена гіпертензії та
дисліпідемії, які добре контролюються ліками. Він заперечує будь-яку епізоди
куріння. При огляді- життєво важливі ознаки стабільні ,гарячка відсутня.
Виявлено блідість кон`юнктиви , лімфаденопатії немає. При пальпації
відчувається збільшення селезінки у лівому верхньому квадранті живота.

Загальний аналіз крові показує:

Кількість лейкоцитів: 14,000 / мм³

Гемоглобін: 9 г / дл

Гематокрит: 30%

Кількість тромбоцитів: 470,000 / мм³

Кілька спроб пункції кісткового мозку були безуспішними. Показано мазок


периферичної крові.

Який з наступних процесів пов'язаний із станом цього пацієнта?

A. Апластична анемія

B. Аспленія

C. Злипання гемоглобіну

D. Внутрішньосудинний гемоліз

E. *Мієлофіброз
4.

4. A 34-year-old woman goes to her physician because of an unexplained 7-kg weight


loss and difficulty sleeping due to excessive sweating for the past 6 months.On
review of systems, the patient notes chest pain, cough, and shortness of breath.After a
chest X-ray shows a large mediastinal mass, a biopsy reveals mononucleate and
binucleate tumor cells in a background of lymphocytes.

Which of the following types of microorganism is most commonly associated with


this condition?

A.Acid-fast bacteria

B. Double-stranded DNA reverse transcriptase virus

C.*Double-stranded virus

D.Papovavirus

E.Retrovirus

4. . 34-річна жінка звернулась до свого лікаря через раптове схуднення на 7-


кілограмів і проблеми зі сном через надмірну пітливість протягом останніх 6
місяців. При огляді пацієнт відзначає біль у грудях, кашель і задуха . Після
рентгенографії грудної клітки виявляється велике новоутворення середостіння,
біопсія виявляє одноядерні і двоядерні пухлинні клітини на тлі лімфоцитів.

Який з наступних типів мікроорганізмів найчастіше викликає такий стан?


A. Кислотостійкі бактерії

B.Дволанцюгові зворотньо-транскипційні ДНК-віруси

C.*Дволанцюгові вірус

D.Papovavirus

E.Retrovirus
5.

5. .A 12-year-old girt is being treated with cyclophosphamide for juvenile rheumatoid


arthritis.She is taking the standard dose of cyclophosphamide and appears to be
hypersensitive to its adverse effects, quickly developing hemorthagic cystitis.On
physical examination, her pediatrician notices scattered petechiae over the child's
body.The doctor also learns that both her paternal uncle and her maternal aunt were
diagnosed at a young age with bone problems.

A complete blood cell count shows:

WBC count: 3,3 K/µL

Hemoglobin: 6,5 g/dl

Hematocrit : 19,5 %

Mean cell volume 116,6 fL

Platelet count: 76 K/µL

Which of the following is the underlying reason for this patient's lab values?

A.Ataxia-telangiectasia

B.*Fanconi anemia

C.Hemophilia A

D.Idiopatic thrombocytopenic purpura


E. von Willebrand disease

5. 12-річний дівчинка проходить лікування Циклофосфамідом від ювенільного


ревматоїдного артриту. Вона приймає стандартну дозу Циклофосфаміду і ,
виявляється, має підвищену чутливість до його побічних ефектів, швидко
розвивається геморагічний цистит. Лікар також дізнається, що і у її дядька, і її
тітки у молодому віці діагностували проблеми кісток.

Загальний аналіз крові показує:

Кількість лейкоцитів: 3,3 К / мкл

Гемоглобін: 6,5 г / дл

Гематокрит: 19,5%

Середній об'єм клітин 116,6 фл

Кількість тромбоцитів: 76 К / мкл

Яке з наведених причин є основною причиною лабораторних показників цього


пацієнта?

A.Атаксия-телеангіектазія

B.*Анемія Франконі

C.Гемофілія А

D.Ідіопатична тромбоцитопенічна пурпура

E. хвороба фон Віллебранда


6.

A 45-year-old woman is brought into the emergency department by her husband, who
reports she has had a fever for the past day and has become lethargic.The patient is
disoriented, with a temperature of 38.3'C and sublingual jaundice. Laboratory tests
show:

Hemoglobin: 8 g/dL

Platelet count: 30,000/mm³

Creatinine: 22 mg/dL

Blood urea nitrogen: 29 mg/dL

Lactate dehydrogenase: 350 u/L

Total bilirubin:2.8 mg/dl


Indirect bilirubin: 2.0 mg/dL

Peripheral blood smear is shown.

Which of the following is the most likely underlying cause of this patient's condition?

A. Antibodies directed against platelets

B. *Decreased activity of von Willebrand factor protease

C. Inherited disorder of hemoglobin

D. Mutations affecting the erythrocyte membrane

E. Viral infection of hepatic cells

6. 45-річну жінку привіз у відділення невідкладної допомоги її чоловік, який


повідомляє, що у неї була гарячка протягом останнього дня і відчуття апатії .
Пацієнтка дезорієнтована, з температурою 38,3 ° С і під'язикової жовтяницею .
Лабораторні тести показують:

Гемоглобін: 8 г / дл

Кількість тромбоцитів: 30,000 / мм³

Креатинін: 22 мг / дл

Азот сечовини в крові: 29 мг / дл

Лактатдегідрогеназа: 350 од / л

Загальний білірубін: 2,8 мг / дл

Непрямий білірубін: 2,0 мг / дл


Показаний мазок периферичної крові.

Яке з наступних причин є найбільш вірогідною причиною такого стану


пацієнтки?

A. Антитіла, спрямовані проти тромбоцитів

B. *Зниження активності протеази фактора Віллебранда

C. Успадковане порушення гемоглобіну

D. Мутації, що впливають на мембрану еритроцитів

E. Вірусна інфекція печінкових клітин


7.

7. A mother brings her 6-year-old son to the clinic to have a large growth on the
child's jaw examined.The boy underwent a heart transplant as infant and had been
doing well until a few months ago when he started to tire easily.He states that the
mass has been slightly uncomfortable, but his mother is mainly worried because it
keeps growing.Results of a biopsy of the mass are shown in the image.
What is the most likely chromosomal translocation that caused his disease?

A.*t(8;14)

B.t(9;22)

C.t(11;14)

D.t(11;18)

E.t(14;18)

7. Мати привела свого 6-річного сина до клініки, щоб оглянули зростаюче


новоутворення на щелепі дитини. Хлопчик переніс пересадку серця ,як був
немовлям, і добре почувався до тих пір, поки кілька місяців тому не почав
легко стомлюватися . Він стверджує, що новоутворення приносить дискомфорт,
але його мати в основному турбується, що пухлина продовжує рости.
Результати біопсії новоутворення зображено на малюнку .

Яка найбільш імовірна хромосомна транслокація спричинила його


захворювання?

A.*t (8; 14)

B.t (9; 22)

C.t (11; 14)


D.t (11; 18)

E.t (14; 18)


8.

8. 19-year-old college student presents to his physician complaining that he has been
extremely lethargic lately.He also mentions that although he drinks plent of water, his
urine has recently looked brown.On physical examination, the patient is found to be
afebrile with mild tachycardia, pale conjunctivae, and yellow- tinged sclerae.The
physician is able to palpate a lump in the left upper quadrant of the patient's
abdomen.The physician orders a full set of laboratory tests and observes the findings
on a peripheral blood smear shown in the image.
Which of the following is the definitive choice of therapy for this patient's condition?

A. Blood transfusion

B. Chemotherapy

C. Folic acid supplementation

D. Hydroxyurea

E. Iron chelation therapy

F. Iron supplementation

G. *Splenectomy.

8. 19-річний студент коледжу скаржиться на те, що був вкрай млявим останнім


часом. Він також згадує, що, хоча він п'є багато води, його сеча нещодавно
виглядала коричневою. При фізикальному обстеженні пацієнта виявлено:
відсутність гарячки, легку тахікардію, бліду кон'юнктиву та жовтуватість склер.

При пальпації лікарем було виявлено новоутворення у лівому верхньому


квадранті живота пацієнта. Лікар призначає повний набір лабораторних тестів і
спостерігає за результатами мазка периферичної крові, показаного на
зображенні.
Яке з наступного є остаточним вибором терапії для стану цього пацієнта?

A. Переливання крові

B. Хіміотерапія

C. Прапарати фолієвої кислоти

D. Гідроксисечовина

E. Лікування хелатами заліза

F. Препарати заліза

Г. *Спленектомія.
9.

9. A 57-year-old man comes for evaluation because of a 4-month history of a tactile


fever, worsening fatigue, and generalized weakness.He states that his clothes fit him
more loosely now than they have in the past.Physical examination reveals enlarged
cervical, supraclavicular, axillary, and inguinal lymph nodes, along with a palpable
spleen and 20-cm lateral lobe of the liver.Lymph node biopsy specimens are sent to
the pathologist.

Which of the following types of neoplastic cells are most common in this patient's
condition?

A.*B lymphocyte

B.Myeloblast

C.Plasma cell

D.Reed-Sternberg cell

E.T lymphocyte

9. 57-річний чоловік прийшов на обстеження через 4-місячну відчутну гарячку ,


прогресуючу стомлюваність та загальну слабкість. Він стверджує, що його
одяг став йому завеликим . Фізичне обстеження виявляє збільшені шийні,
надключичні, пахвові і пахові лімфатичні вузли, разом з тим пальпується
селезінка і 20-сантиметрова латеральна частка печінки.Зразки біопсії
лімфатичного вузла направляються патологоанатому.
Які з наступних типів пухлинних клітин найчастіше зустрічаються в стані цього
пацієнта?

A.*B лімфоцити

B.Мієлобласти

C. Плазматичні клітини

Д.Клітини Ріда-Штернберга

Е.Т лімфоцити
10.

10.
A 62-year-old man is admitted to the hospital for a work-up following a 1-month
history of fatigue and problems with balance . He says that when he gets up in the
middle of the night to go to the bathroom, he often stumbles and bumps into the
wall.Physical examination reveals an unsteady gait and difficulity determining which
toe a vibrating tuning fork is placed on.Ankle jerk reflexes are decreased
bilaterally.Laboratory tests show

Hematocrit : 29 %

WBCs :7100/mm³

Platelets: 240,000/mm³

Mean corpuscular volume: 123 um³

Peripheral blood smear is shown.

Which of the following is the most likely cause of this patient's low hematocrit level?

A.Anemia of chronic disease

B. Autoimmune hemolytic anemia

C. Folate deficiency

D. Hereditary spherocytosis

E. Iron deficiency anemia

F. *Pernicious anemia

10. 62-річного чоловіка госпіталізують на огляд після 1-місячної втоми та


проблем з рівновагою. . Він каже, що коли він встає в середині ночі, щоб піти у
ванну, він часто спотикається і врізається в стіну. Фізичне обстеження виявляє
нестійку ходу і труднощі у визначенні на якому пальці стоїть вібраційний
камертон. Рефлекси гомілковостопного суглобу двосторонньо зменшуються.

Лабораторні тести показують

Лейкоцити: 7100 / мм³


Тромбоцити: 240,000 / мм³

Середній об'єм корпускули: 123 мкм³

Показано мазок периферичної крові.

Яке з наступних причин є найбільш імовірною причиною низького рівня


гематокриту у цього пацієнта?

A.Анемія внаслідок хронічного захворювання

B. Аутоімунна гемолітична анемія

C. Дефіцит фолієвої кислоти

D. Спадковий сфероцитоз

E. Залізодефіцитна анемія

F. *Перніціозна анемія
11.

11. A 74-year-old man presents with weakness and fatigue. He is concerned that, despite no
alteration in his dietary intake, his midsection seems to have expanded in the last few weeks,
leaving his clothes feeling tight around his abdomen. He also remarks that he has recently been
changing his T-shirts in the middle of the night because he wakes up drenched in sweat. His
medical history is significant for emphysema, elevated LDL levels, and Crohn disease. His regular
medications include an ipratropium inhaler, atorvastatin, and 6-mercaptopurine. He has a brother
with T-cell leukemia. He also recently treated his wood deck with a chemical pesticide. Physical
examination demonstrates profound splenomegaly. A biopsy of the enlarged spleen is obtained
(shown in the image), which shows sheets of large lymphocytes that stain positively for CD20.
Which of the following conditions is thought to predispose a patient to this disease?
A. Arsenic exposure
B. Human T-lymphocyte leukemia virus infection
C. *Immunosuppression
D. Mutation in the β-globin gene
E. Smoking

11.74-річний чоловік скаржиться на слабкість і втому. Він стурбований тим, що, незважаючи
на відсутність будь-яких змін в раціоні харчування, окружність його талії збільшилася за
останні кілька тижнів, в результаті чого одяг став щільно прилягати до живота. Він також
зазначає, що нещодавно він міняв свої футболки серед ночі, бо прокидався мокрий від поту.
Його історія хвороби характерна для емфіземи, підвищених рівнів ЛПНЩ і хвороби Крона.
Для лікування він регулярно використовує такі препарати: інгалятор ипратропия,
аторвастатин і 6-меркаптопурин. У нього є брат з Т-клітинним лейкозом. Він також недавно
обработивал дерев'яний настил хімічним пестицидом. Фізикальне обстеження показало
глибоку спленомегалию. Зроблена біопсія збільшеної селезінки (показано на малюнку), на
якій показані скупчення великих лімфоцитів, які фарбуються позитивно для CD20.
Какое из следующих условий считается предрасполагающим пациента к этому заболеванию?

A. Вплив миш'яку
B. Інфекція, спричинена вірусом лейкозу Т-лімфоцитів людини
C. *Імуносупресія
D. Мутація в гені β-глобіну
E. Куріння
12.

12. 64-year-old man comes to his physician because of lethargy over the past several months, along
with a cough producing brownish sputum. He has been treated twice in the past 3 months with
antibiotics for community-acquired pneumonia. Temperature is 39.1'C (102.4'F). Physical
examination discloses right sided pulmonary crackles, nontender cervical and axillary
lymphadenopathy, and hepatosplenomegaly. Laboratory studies show
WBC count: 40,000/mm3
Hemoglobin: 10 g/dL
Platelet count: 190,000/mm3
Neutrophils : 20 %
Lymphocytes : 77 %
Monocytes : 1.5 %
Eosinophils : 1 %
Basophils : 0.5 %
An image of his blood smear is shown below.

Which of the following conditions is most commonly associated with this patient's disease?
A. *Autoimmune hemolytic anemia
B. Autoinfarction of the spleen
C. Disseminated intravascular coagulation
D. Fungating skin lesions
E. Lytic bone lesions
F. Progression to blast crisis

12. 64-річний чоловік звернувся до свого лікаря через апатії протягом останніх кількох
місяців, а також кашлю з коричневою мокротою. За останні 3 місяці його двічі лікували
антибіотиками від пневмонії. Температура становить 39,1 ° С (102,4 ° F). Фізикальне
обстеження виявило правосторонні легеневі хрипи, безболісну шийну і пахвову
лімфаденопатія, і гепатоспленомегалію. Лабораторні дослідження показують
Рівень лейкоцитів: 40,000/mm3
Гемоглобін: 10 g/dL
Количество тромбоцитов: 190,000/mm3
Нейтрофіли: 20%
Лімфоцити: 77%
Моноцити: 1,5%
Эозинофіли: 1%
Базофіли: 0,5%
Нижче показано зображення мазка крові пацієнта.

Яке з наступних умов найімовірніше пов'язано з хворобою цього пацієнта?


A. *Аутоіммунна гемолітична анемія
B. Аутоінфаркт селезінки
C. Розсіяне внутрішньосудинне згортання
D. Грибкові ураження шкіри
E. Літичне ураження кісток
F. Прогресування бластного кризу
13.

13. A 23-year-old woman presents to the clinic because of persistent fatigue, fever, and pain. She
reports losing 15 lb in the past 3 months and has had pain in various joints in her wrists and hands,
which prevents her from being able to work. The patient is also concerned because she has had
multiple miscarriages in the past 3 years. Initial laboratory tests show a positive rapid plasma reagin
(RPR) test result and a negative fluorescent treponemal antibody (FTA) test result.
Which of the following additional laboratory findings is most likely in this patient?
A. Decreased partial thromboplastin time, corrected by mixing 1:1 with fresh-frozen plasma
B. Decreased partial thromboplastin time, not corrected by mixing 1:1 with fresh-frozen plasma
C. Prolonged partial thromboplastin time, corrected by mixing 1:1 with fresh-frozen plasma
D. *Prolonged partial thromboplastin time, not corrected by mixing 1:1 with fresh-frozen plasma

13. 23-річна жінка прийшла в лікарню через постійну втому, лихоманку і боль. Вона
повідомляє, що втратила 15 фунтів за останні 3 місяці, скаржиться на болі в різних суглобах
на зап'ястях і кистях, що не дозволяє їй працювати. Пацієнтка також стурбована тим, що у
неї було багато викиднів за останні 3 роки. Початкові лабораторні аналізи показують
позитивний результат тесту на швидкий плазмовий реагіни (ЕДС) і негативний результат
тесту на флуоресцентні трепонемні антитіла (РІФ).
Які з наступних додаткових лабораторних результатів найбільш вірогідні у цього пацієнта?
A. Зменшена частковий тромбопластиновий час, скориговане шляхом змішування 1: 1 зі
свіжозамороженої плазмою
B. Зменшена частковий тромбопластиновий час, не скориговане шляхом змішування 1: 1 зі
свіжозамороженої плазмою
C. Збільшене частковий тромбопластиновий час, скориговане шляхом змішування 1: 1 зі
свіжозамороженої плазмою
D. *Збільшене частковий тромбопластиновий час, не скориговане шляхом смешеванія 1: 1 зі
свіжозамороженої плазмою
14.

14. A 70-year-old woman presents to the emergency department after having fallen while walking
in her house. She is holding her right thigh, stating the pains 10/10 and sharp, leaving her unable to
move the leg or put pressure on it. On review of systems, the patient notes that over the past several
weeks, she has had shortness of breath, feelings of lethargy, weakness, and occasional bone pain.
The physician orders lab tests, which produce the following results:
Hemoglobin: 7g/dL
Hematocrit : 21%
Reticulocyte count : 0.1 % ( normal 0.5-1.5% )
Platelet count: 50,000/mm3
Leukocytes: 1,300/ mm3
Neutrophils: 44%
Eosinophils : 2%
Lymphocytes : 46%
Monocytes : 6%
24-hr Urine collection: 4g of protein (normal <1)
Calcium: 13.1mg/dl
Which of the following findings is most likely in this patient?
A. Broken and destroyed lymphocytes
B. Increased levels of circulating promyelocytes and myelocytes
C. Microcytosis and hypochromasia
D. Myeloblasts with Auer rods
E. *Rouleaux formation

14. 70-річна жінка надійшла у відділення невідкладної допомоги після того, як впала,
гуляючи по дому. Вона тримається за праве стегно, зазначаючи, що біль 10/10 і гостра, і вона
не може поворухнути ногою або наступити на неї. В огляді систем пацієнтка відзначає, що
протягом останніх декількох тижнів у неї були задишка, відчуття втоми, слабкості і
випадкові болю в кістках. Лікар призначає лабораторні аналізи, які дають такі
результати:Гемоглобин: 7 г/дл
Гематокрит: 21%
Кількість ретикулоцитів: 0,1% (нормальное 0.5-1.5%)
Кількість тромбоцитів: 50,000 / мм3
Лейкоцити: 1,300 / мм3
Нейтрофіли: 44%
Эозинофіли: 2%
Лімфоцити : 46%
Моноцити: 6%
24-годинний сбір сечі: 4 г білку (в нормі <1)
Кальцій: 13,1 мг/дл.
Які з наступних результатів найбільш вірогідні у цього пацієнта?

A. Пошкоджені і зруйновані лімфоцити


B. Підвищений рівень циркулюючих промієлоцитів і мієлоцитів
C. Мікроцитоз і гіпохромазія
D. Мієлобласти з тільцями Ауера
E. *Утворення "монетних стовпчиків"
15.

15. A 6-year-old boy is brought to his pediatrician by his mother who says that he has had a fever
during the past week. She also says the child has been tired and weak. Physical examination shows
hepatosplenomegaly, generalized lymphadenopathy, and pale conjunctivae. Laboratory studies
show a hemoglobin of 7.1 g/dl and a platelet count of 53,000/mm 3. Additionally, the peripheral
blood shows atypical clls which are also found in a bone marrow aspirate. These cells stain
positively for CD10, CD19, and CD20. Cytogenetic analysis of this population reveals a t(12:21)
rearrangement.
Which of the following is the most likely diagnosis?
A. *Acute lymphoblastic leukemia
B. Acute myeloid leukemia
C. Chronic lymphocytic leukemia
D. Iron deficiency anemia
E. Scarlet fever

15. 6-річного хлопчика привела до педіатра його мати зі скаргами на лихоманку протягом
останнього тижня. Вона також каже, що дитина втомлений і слабкий. Фізикальне обстеження
показало гепатоспленомегалію, генералізована лімфаденопатія і бліду кон'юнктиву.
Лабораторні дослідження показують, що гемоглобін становить 7,1 г / дл, а кількість
тромбоцитів - 53 000 / мм 3. Крім того, в периферичної крові виявляються атипові клітини,
які також виявляються в пунктаті кісткового мозку. Ці клітини позитивно фарбуються на
CD10, CD19 і CD20. Цитогенетичний аналіз цієї популяції виявляє перегрупування t (12:21).
Який з наступних діагнозів є найбільш імовірним?
A. *Гострий лімфобластний лейкоз
B. Гострий мієлоїдний лейкоз
C. Хронічний лімфобластний лейкоз
D. Залізодефіцитна анемія
E. Скарлатина
16.
16. A 40-year-old woman presents to the emergency department in a confused state, with fever and
jaundiced eyes. Laboratory tests show:
Totat bilirubin: 3.6 mg/dL
Direct bilirubin: 0.2 mg/dL
Hematocrit : 28 %
LDH: 650 U/L
PT: 13 sec
PTT: 32 sec
A peripheral blood smear is shown.

Which of the following laboratory abnormalities would also be expected in this patient?
A. Decreased blood urea nitrogen level
B. Decreased phosphorus level
C. Decreased potassium level
D. Elevated bicarbonate level
E. *Elevated creatinine level

16. 40-річна жінка надходить у відділення невідкладної допомоги з спутаним свідомістю,


лихоманкою і жовтяничними очима. Лабораторні тести показують:
Загальний білірубін: 3,6 мг / дл.
Прямий білірубін: 0,2 мг / дл
Гематокрит: 28%
ЛДГ: 650 U/L
ПТЧ: 13 сек
АЧТЧ: 32 сек
Зображено мазок периферичної крові.
Які з наступних лабораторних відхилень також слід очікувати у цього пацієнта?

A. Знижений рівень азоту сечовини в крові

B. Знижений рівень фосфору

C. Знижений рівень калію:

D. Підвищений рівень бікарбонату

E. Підвищений рівень креатиніну


17.

17.A 35-year-old man, with no history of chronic disease and no family history of hospitalization,
comes to the emergency department with fever, neck stiffness and pain behind his eyes. He says he
was taken to a trauma center in rural Ecuador 2 years ago after a motor vehicle accident, that caused
blunt trauma to the abdomen that required surgery to stop internal bleeding He was due for a
follow-up appointment to receive vaccinations but flew home before the appointment. He has not
seen a doctor since. Which of the following findings would likely be seen in a peripheral blood
smear from this patient?
A. Hypersegmented neutrophils
B. Nucleated erythrocytes
C. Schistocytes
D. Sickle cells
E. *Target cells

17. 35-річний чоловік, у якого немає хронічних захворювань і сімейного анамнезу


госпіталізації, надходить у відділення невідкладної допомоги з лихоманкою, регідностю
потиличних м'язів і болем в очах. Він каже, що його доставили до травматологічного центру
в сільському Еквадорі 2 роки тому після автомобільної аварії, яка призвела до тупої травми
живота, яка потребувала хірургічного втручання, щоб зупинити внутрішню кровотечу. Він
повинен був пройти повторний прийом, щоб отримати щеплення, але полетів додому до
призначення. З тих пір він не відвідував лікаря. Які з наступних результатів можуть бути
видні в мазку периферичної крові у цього пацієнта?
A. гіперсегментовані нейтрофіли
B. Ядерні еритроцити
C. Шистоцити
D. Серповидні клітини.
E. *Клітини-мішені
18.

18. A 61-year-old woman complains of a tingling sensation in her feet that has become
progressively worse over the past several months. The patient works as a pastry chef in New York
City and has not traveled anywhere outside of the city in the last 5 years. On physical examination,
she appears mildly jaundiced, and her tongue has a glazed appearance. Neurologic findings include
decreased vibration sense and decreased muscle strength in her lower extremities A peripheral
blood smear is shown in the image
Which of the following is the most likely cause of this patient's disorder?
A. Bacteria overgrowth
B. Dietary deficiency
C. Gastrectomy
D. Infection with Diphyllobothrium latum
E. Pancreatic insufficiency
F. *Pernicious anemia

18. У 61-річної жінки спостерігається поколювання в ногах, яке поступово погіршується за


останні кілька місяців. Пацієнтка працює шеф-кондитером в Нью-Йорку і за останні 5 років
не виїжджала за межі міста. При фізикальному огляді вона виглядає злегка жовтушною, а
язик має глянцевий вид. Неврологічні дані включають зниження чутливості до вібрації і
зниження м'язової сили в нижніх кінцівках. На знімку показаний мазок периферичної крові.

Що з перерахованого є найбільш ймовірною причиною порушень у цього пацієнта?


A. Надмірне розвиток мікрофлори
B. Нестача поживних речовин
C. Гастректомія
D. Інфікування Diphyllobothrium latum
E. Недостатність підшлункової залози
F. *Перніциозная анемія
19.

19.A 67-year-old man presents with a 6-month history of increasing weakness, gingival bleeding,
and weight loss. The patient also complains that his arms and legs are easily bruised. Physical
examination reveals widespread ecchymosis, a palpable spleen tip, and enlarged liver, along with
fixed and enlarged cervical, axillary, and inguinal lymph nodes. A peripheral blood smear shows
increased numbers of damaged lymphocytes. A CBC with differential is performed, and the results
are as follows:
Hemoglobin:9.0g/dL
Hematocrit : 38 %
WBC: 10,500/mm3
Platelets: 100,000/mm3
Neutrophils : 20 %
Lymphocytes: 70%
Monocytes : 8%
Eosinophils : 1.5 %
Basophils : 0.5 %
This patient's disease process involves a clonal expansion of which type of cell?
A. *B lymphocyte
B. Metamyelocyte
C. Plasma cell
D. Promyelocyte
E. T lymphoblast

19.67-річний чоловік має 6-місячний анамнез: наростаюча слабкість, кровоточивість ясен і


втрата ваги. Пацієнт також скаржиться на те, що на руках і ногах легко утворюються синці.
Фізикальне обстеження виявляє широко поширений екхімоз, пальпуємий кінчик селезінки і
збільшену печінку, а також нерухомі і збільшені шийні, пахвові і пахові лімфатичні вузли.
Мазок периферичної крові показує збільшення кількості пошкоджених лімфоцитів.
Проводиться ОАК з підрахунком формених елементів, результати наступні:
Гемоглобін: 9,0 г/ д
Гематокрит: 38%
Рівень лейкоцитів: 10 500 / мм3
Тромбоцити: 100 000 / мм
Нейтрофіли: 20%
Лімфоцити: 70%
Моноцити: 896
Еозинофілы: 1,5%
Базофіли 0,5%

Хвороба цього пацієнта включає клональную експансію клітин якого типу?


А. *B-лімфоцит
Б. Метамієлоцити
С. Плазматична клітина
Д. Проміелоціт
Е. Т-лімфобластів
20.

20. A frail 71-year-old female presents to the emergency department with fatigue, dizziness, and
palpitations. The patient used to eat regularly, but her daughter reports that since her mother was
widowed 5 years ago, she frequently skips meals now that she lives alone. When she does eat, her
meals consist of small quantities of coffee, rice crackers, and jelly. A blood draw reveals the
following results
RBC:4.3 102/L
Hgb: 10.9 g/dL
MCV: 105.9 fL
This patient most likely has a defect in which process?
A. Glutathione synthesis
B. Heme synthesis
C. Isomerization of methylmalonyl-CоA
D. *One-carbon-unit transfers
E. Synthesis of β-globin chains
F. Synthesis of spectrin-actin cytoskeleton

20.Слабка 71-річна жінка надходить у відділення невідкладної допомоги з втомою,


запамороченням і серцебиттям. Пацієнтка регулярно харчувалася, але її дочка повідомляє,
що, оскільки її мати овдовіла 5 років тому, вона часто пропускає прийоми їжі тепер, коли
вона живе одна. Коли вона їсть, її їжа складається з невеликої кількості кави, рисових
крекерів і желе. Забір крові показує наступні результати:
Кількість еритроцитів:4.3 1012/L
Hb: 10.9 g/dL
Середній об’єм єритроцитів: 105.9 fL
Цей пацієнт, швидше за все, має порушення в процесі

A. Синтез глутатіону

B. Синтез гема

C. Ізомеризація-КоА

D. *Перенесення одноуглеродних фрагментів

E. Синтез ланцюгів β -глобіна

F. Синтез спектрин-актинового цитоскелета


21.

21.6-річний греко-грецький родовід потрапив до лікарні через дегідратацію внаслідок спазмового


болю в животі і кривавої діареї. Навчальний педіатр побачив дитину 1 тиждень тому для перевірки
дитини, коли педіатр дізнався, що дитина живе в старому будинку Після останнього візиту у дитини
з'явилася діарея, яку мама спробувала лікувати антибіотики призначають їй 4 роки тому. Діарея
продовжувалася, незважаючи на лікування. Покинувши результати культури калу для визначення
терапії, повертаються результати лабораторних досліджень, які включають наступне:

Кількість лейкоцитів: 16 000 / мм

Кількість еритроцитів: 4,3 x 1012 / л

Гемоглобін: 10,9 г / дл

Гематокрит: 33,2%

Середній корпускулярний обсяг: 86,5 мкл

кількість тромбоцитів: 90 000 / мм 3

Креатинін: 8 , 4 мг / дл

Показані лабораторні дані в периферичної крові.

Який процес міг би пояснити результати мазка периферичної крові?

A. Антитіло-опосередковане руйнування змінених еритроцитів

B. Зниження активності глюкозо-6-фосфатдегідрогенази

C. Інгібування синтезу гему

D. Нестабільність мембран RBC, викликаних внутрішнім дефектом

E. *Серпування еритроцитів внаслідок аномального гемоглобіну

F. Хвороби еритроцитів внаслідок аномального гемоглобіну


22.

22. 28-річна біла жінка показує своєму лікарю набряк і біль у правій нозі, що почалася 2 дні тому.
Вона каже, що у неї немає медичних проблем, але повідомляє про випадкові болі в суглобах її рук, а
також про періодичні афтозні виразки. Вона не приймає ніяких ліків. Фізичне обстеження виявляє
інтактний пульс dorsalis pedis, а також біль на дорсифлексию правої стопи. На її обличчі відзначається
висип (показано на малюнку). Коли її запитують про її висип на обличчі, вона каже, що вона була там
протягом кількох років і погіршується, коли вона виходить на сонце. Ультразвукове дослідження
правої ноги виявляє невключний тромб в правій підколінної вені. Результати лабораторних
досліджень такі:

гемоглобін: 10,1 г / дл

білірубіну (всього) 0,8 мг / дл.


Креатинін: 14 мг / д.

Кількість тромбоцитів: 100000 / мм.

Протромбіновий час: 24 секунди

Час часткового тромбопластину: 89 секунд

Яка з наведених причин є найбільш імовірною причиною стану цього пацієнта?

A. Антитіла, спрямовані проти RBCS

B. Антитіла, спрямовані проти фактора Vlll

C. Антитіла, спрямовані проти гепарину

D. Антитіла, спрямовані проти глікопротеїну тромбоцитів llb / IIla

E. *Антитіла проти фосфоліпідів тромбоцитів


23.

23. 15-річна афро-американська дівчина подарує відділення невідкладної допомоги після декількох
днів лихоманки і болю, які, за її словами, зробили її все важче ходити. Пацієнтка має історію
раптового початку болю в кістках і суглобах. Вона також мала болючі і розпухлі руки і ноги,
двосторонньо. Фізичне обстеження виявляє болючість і набряклість її стегон. Повний аналіз крові
(CBC) показує лейкоцитоз 15000 WBC / мм2. Висновки на звичайній плівці і МРТ узгоджуються з
остеомієлітом праксимальної стегнової кістки. Показано мазок периферичної крові
Грам фарбування найбільш ймовірного інфекційного агента в цьому випадку покаже, що з
наступного?

A. Грамнегативні диплококи

B. *Грамнегативні палички

C. Грампозитивні коки в ланцюгах

D. Грампозитивні коки в кластерах

E. Грампозитивні стрижні
24.

24. 33-річний недавній іммігрант представляє відділення невідкладної допомоги з історією


підвищення температури і втоми протягом останніх декількох тижнів. Спостерігається велика маса
лівої нижньої щелепи. Біопсія маси показана на зображенні
Який ген надмірно експресується в цих клітинах?

A. BCL-2

B. BCR-ABL

C. IL-21

D. TdT

Е. *C-myc
25.
25. 63-річна жінка приходить до офісу через втому, яка погіршувалася за останні 2 тижні. Спочатку
вона думала, що це "частина процесу старіння", але потім почала помічати "плями" і синці на її шкірі і
губах. Вона заперечує біль у кістках. Її температура становить 37,4 С (99,3F), артеріальний тиск 135/80
мм рт.ст., а дихання 16 / хв. Пацієнт має блідий вигляд. Її звуки дихання ясні двосторонні, а серцеві
звуки нормальні. Немає збільшених лімфатичних вузлів. Живіт не є сприйнятливим, а
гепатоспленомегала немає. Повний аналіз крові виявляє наступне: Гемоглобін: 8 г / дл Тромбоцити:
67000 / мм Лейкоцити: 3000 / мм Міолабласти: 30%. Проміелоцити: 10% Мієлоцити: <5%
Метаміелоцити: <5% Зрілі мієлоцити: 50% Показано мазок периферичної крові.

Яке з перерахованих нижче випадків, швидше за все, пов'язане зі станом цього пацієнта?

A. Ураження доменних кісток на рентгенографії нижніх кінцівок

B. Зниження активності фактора фактора Віллебранда в плазмі

C. Більше 20% лімфобластів у кістковому мозку

D. Підвищена лужна фосфатаза лейкоцитів

E. *Мікроангіопатична гемолітична анемія


26.

26. 2-річну дівчинку привозять до педіатра матір'ю для оцінки 2-денної історії збільшення
дратівливості, лихоманки і червоної області на лівій нозі, що збільшується в розмірах. У неї були м'які
кишечники і три мокрих підгузника на день, вона не блювала. Мати каже, що дитина легко отримує
синці і мав дві інфекції вуха в минулому році і одну інфекцію синуса минулого місяця. вага Життєві
ознаки показують частоту серцевих скорочень 110 / хв, кров'яний тиск 100/60 мм рт.ст., частоту
дихання 30 / хв і температуру 101,4F (38,5 С). При фізичному огляді пігментація шкіри легка, а волосся
розріджені і блондинка. Учні становлять 2 мм і реагують на світло, а іриси блакитні. На лівому стегні
дитини трохи піднятий, еритематозний і набряклий ділянку на лівому стегні дитини, що ніжно на
дотик і дренує гнійну рідину. Інша частина фізичного обстеження є нормальною.

A. *Синдром Chediak Higashi

B. Хронічне гранулематозне захворювання

C. Синдром гіпер-lgM

D. Селективний дефіцит IgA

E. Важкий комбінований імунодефіцит


27.

27. 3-річний хлопчик потрапляє в відділення швидкої допомоги з сильним болем у правому колі. Його
мати повідомляє, що її син скаржився на подібний біль у іншому коліні 4 місяці тому, хоча це було не
так важко. Огляд систем виявляє інфекцію верхніх дихальних шляхів 6 місяців тому, лікувався за
допомогою ліків без рецепта. При огляді коліно набрякло, а дитина відмовляється рухати суглоб.
Немає еритеми або теплоти над ураженим суглобом, але екхімози відзначаються за двосторонніми
ліктями Лабораторні дослідження показують, що:

гематокрит: 44%

Кількість тромбоцитів: 150,000 / мм


Лейкоцити 9300 мм

Нейтрофіли: 64%

Еозинофіли: 2%

PTT: 80

INR: 1.1

Який із наведених механізмів, швидше за все, відповідає презентації цього пацієнта?

A. *Зниження фактора VIll

B. Зниження фактора Віллебранда

С. Екстенсивна травма

D. Імунна деструкція тромбоцитів

E. Колонізація золотистого стафілокока


28.

28. 61-річний чоловік приходить до офісу через 3-місячну історію втоми. Він має історію розладу
вживання алкоголю і цукрового діабету 2 типу. Він також повідомляє про втрату ваги, зниження
апетиту, легкий синці і розмиті болі в кістках. Він заперечує нічну пітливість, кашель крові, зміни зору,
біль у грудях, задишку або біль у животі. Його життєві ознаки знаходяться в межах норми. У його
склерах з'являються жовтяниці і слизові оболонки з'являються бліді. Його легені ясні для аускультації
двосторонньо і частота серцевих скорочень регулярна без шумів, його живіт м'який і безболісний.
Край печінки пальпується на 3 см нижче правого краю і пальпується кінчик селезінки. Показано мазок
периферичної крові.
Яке з перерахованих нижче факторів, швидше за все, відповідає за порушення печінки цього
пацієнта?

А. Накопичення жиру в гепатоцитах

В. Автоімунне руйнування внутрішньопечінкових жовчних проток

C. *Екстрамедуллярний кровотворення

D. Набряк і некроз гепатоцитів

Е. Метастатична інфільтрація печінки


29.

29. 7-річного хлопчика доставляють до свого педіатра з 3-тижневою історією лихоманки і слабкості. і
мазок периферичної крові, який виявляє наступне:

гемоглобін: 7,2 г / дл

лейкоцитів: 6100 / мм

тромбоцитів: 49 000 / мм

Які з наступних умов найбільш тісно пов'язані з можливим діагнозом цього пацієнта?
A. *Синдром Дауна

B. Інфекція Епштейна-Барра

C. ВІЛ-інфекція

D. Імунна тромбоцитопенічна пурпура

Е.Синдром Тернера
30.

30. 45-річний водій вантажівки відвідує свого лікаря первинної медичної допомоги для щорічного
обстеження. Пацієнт згадує, що протягом останніх 3 місяців було чотири грипоподібні епізоди. Він
стверджує, що останнім часом відчуває себе слабким і що він збільшив дозу напроксена, щоб
«боротися з болями». Він каже, що за останні 3 місяці біль у спині збільшився, і він продовжує сидіти
протягом тривалого часу. Незважаючи на те, що він п'є 2 літри води на день, його сечовипускання
зменшилося за останні кілька місяців. Лікар замовляє різні тести, включаючи зображення і аналізи
крові. Показані результати аналізу крові

Яка найбільш імовірна причина ниркової недостатності пацієнта?

A. Реакція гіперчутливості II типу

B. Гострий пієлонефрит
C. Осадження IgM в субендотеліальному просторі

D. Обструкція неопластичною масою

Е. *Виробництво великих трубчастих виливків


31.

31. 28-річна жінка потрапляє в відділення екстреної допомоги через біль і набряклість у правому
телятині. Вона каже, що біль почався раптово кілька годин тому після того, як вона висадилася з
повернення з Японії. На оцінці сьогодні пацієнт виявляється нервовим і швидко дихає. Вона
розповідає лікареві, що вона вважає, що її симптоми пов'язані з їжею pufferfish як останньої їжі в
Японії. Пацієнт афебрильний. Її артеріальний тиск становить 116/77 мм рт.ст., пульс - 107 / хв, частота
дихання - 20 / хв, насичення киснем - 94%. Під час респіраторного дослідження вона повідомляє про
біль на глибокому натхненні. Її ноги опухлі двосторонньо до середини теляти; у пацієнта
спостерігається нелітеральний біль у литки при пальпації.

У цієї жінки, швидше за все, є які з наступних основних розладів?

A. Дефіцит антитромбіну III

B. *Фактор V Leiden

C. Дефіцит білка C

D. Дефіцит білка S

E. Мутація гена протромбіну


32.

32. 20-річна жінка подарує гінекологу через 7-місячну гістологію тривалих важких періодів.
Лабораторні тести показують:

PT: 13,1 секунди

INR: 1,1

PTT: 46 секунд

Кількість тромбоцитів: 250,000 / мм3

Подальше тестування показує, що рівні фактора VIII і фактора Вілллебранда (vWF) аномально низькі.
Втрата якої специфічної функції vWF відповідає за спостережувані лабораторні відхилення?

A. Активація фактора X

B. Активація фактора X поряд з тканинним фактором

C. Адгезія тромбоцитів

D. Білок-носій для фактора IX

E.* Білок-носій для фактора VIII


33.

33. A 30-year-old woman comes to the emergency department because of shortness ob breath. The patient
was at her desk at work when she began experiencing sharp, localized pain, which she states was made
worse by deep inspiration. The patient looks very distressed and cannot speak in complete sentences. She
denies taking any medications, including oral contraceptives. Physical examination is also notable for
erythema, edema, and tenderness to deep palpation in her right leg. On evaluation, her temperature is
103,1 F(39,5C) , pulse is 110/min, respiratory rate is 22/min, and blood pressure is 135/70 mm Hg. Her
laborotory test result are as follows:

Hematocrit: 44%

Platelet count: 250,000/mm3

Leucocytes: 20,000/mm3
Neutrophils: 58%

Eosinophils: 2%

Lymhpocytes: 34%

Monocytes: 6%

Fibrinogen: 200mg/dL

INR: 1,1

D-dimer: 500ng/mL (normal<250 ng/mL)

Which of the following is the most common condition predisposing this patient to her current
presentation?

A. A genetic defect that causes the prothorbin level to increase

B. *Autosomal dominant condiiton preventing protein C from degrading factor V

C. Genetic absence of antithrombin III

D. Genetically abnormal fibrinogen causing clots

E. Protein deficiency resulting in increased factor Va and VIIIa

33. 30-річна жінка приїжджає до відділення невідкладної допомоги через задишку. Пацієнтка
перебувала за робочим столом, коли почала відчувати гострий, локалізований біль, який, за її
словами, погіршувала глибока натхнення. Пацієнт виглядає дуже засмученим і не може говорити
повним реченням. Вона заперечує прийом будь-яких ліків, включаючи оральні контрацептиви.
Фізичне обстеження також помітне еритемою, набряком і болючістю до глибокої пальпації в правій
нозі. При оцінці її температура становить 103,1 F (39,5 ° C), пульс 110 / хв, частота дихання 22 / хв,
кров'яний тиск 135/70 мм рт. Її результат лабораторного тесту наступний:

Гематокрит: 44%

Кількість тромбоцитів: 250,000 / мм3

Лейкоцити: 20,000 / мм3

Нейтрофіли: 58%

Еозинофіли: 2%

Лімфоцити: 34%

Моноцити: 6%

Фібриноген: 200 мг / дл

INR: 1,1

D-димер: 500 нг / мл (нормальний <250 нг / мл)

Яке з наведеного нижче є найпоширенішим умовою, що схиляє цього пацієнта до її поточної


презентації?
A. Генетичний дефект, що викликає підвищення рівня проторбіну

B. *Аутосомно-домінантний кондиціон, що запобігає протеїну С фактора деградації V

C. Генетична відсутність антитромбіну III

D. Генетично патологічний фібриноген, що викликає згустки

E. Дефіцит білка призводить до збільшення фактора Va і VIIIa


34.

34. A 14-year-old girl is brought to the physician by her mother after the girl experiences multiple episodes
of severe shortness of breath during gym class at school. Her mother notes that she looks paler than usual.
There is no history of menorrhagia. On examination, vital sings are stable. There is no lymphadenopathy or
hepatosplenomegaly. Complete blood count shows mild anemia. Coombs test is negative; her blood smear
is shown.
Which of the following is the most likely pathogenesis of this dirl's anemia?

A. Glucose-6-phosphate dehydrogenase deficiency

B. Mutation in the phosphatidylinositol glycosyltransferase gene

C. *Mutation in the spectrin gene

D.Physical disruption of otherwise normal RBCs

E. Point mutations of one of the B-globin genes

34.14-річну дівчинку привозять до лікаря матір після того, як дівчина відчуває кілька епізодів важкої
задишки під час занять у тренажерному залі в школі. Її мати зазначає, що вона виглядає блідішою за
звичайну. Історії меноррагії немає. При обстеженні життєві співи стабільні. Відсутня лімфаденопатія
або гепатоспленомегалія. Повний аналіз крові показує легку анемію. Тест Кумбса негативний;
показаний мазок крові.

Який з наступних є найбільш імовірним патогенезом анемії цієї дитини?

A. Дефіцит глюкозо-6-фосфатдегідрогенази

B. Мутація гена фосфатидилинозитола глікозилтрансферази

C. *Мутація в гені спектрину

D.Фізичне порушення нормального RBC

E. Точкові мутації одного з генів В-глобіну


35.

35. On newborn screening, an infant is noted to have a midly low hemoglobin concentration. Hemoglobin
electrophoresis testing at 6 months shows the following:

HbA1:53%

HbA2:2%

HbF: 5%

HbS:40%

HbC:0%
The parents are concerned and want to know what this will mean for their child. The parents should be
counseled that their child's disorder is most commonly associated with which of the following
complications?

A. Acute chest syndrome

B. Aplastic anemia

C. Autosplenoctomy

D. Dactylitis

E. *Episodic hematuria

F. Stroke

35.При новонародженому скринінгу відзначається низька концентрація гемоглобіну. Тестування


електрофорезу гемоглобіну через 6 місяців показує наступне:

HbA1: 53%

HbA2: 2%

HbF: 5%

HbS: 40%

HbC: 0%

Батьки стурбовані і хочуть знати, що це буде означати для їхньої дитини. Батькам слід порадити, щоб
розлад їхньої дитини найчастіше був пов'язаний з якими з таких ускладнень?

A. Гострий синдром грудної клітки

B. Апластична анемія

C. Автоспенэктомія

D. Дактиліт

E. *Епізодична гематурія

F. Інсульт
36.

36.A 43-year-old man comes to his physician because of fatigue. The patient is concerned because he has a
strong family history of cancer. He thinks that smoking, drinking, and a poor diet prevented his parents
from living longer lives. Because of all this,he never drinks alcohol or smokes tobacco. He has also followed
a strict vegan dietfor 10 years. All of his meals are high in leafy green vegetables. Laborotory tests show a
hematocrit of 35% with a normal RBC distribution width. Results of a peripheral blood smear are shown in
the image.
Which of the following will most definitively determine the likely cause of the patient's fatigue?

A. Folute level

B. Homocysteine level

C.*Methylmalonic acid level

D. Serum gastrin levels

E. Urine vitamin B12 level

36. 43-річний чоловік приходить до свого лікаря через втома. Пацієнт занепокоєний тим, що має
сильну сімейну історію раку. Він вважає, що куріння, вживання алкоголю та погане харчування не
дозволяли батькам жити довше. Через це він ніколи не п'є алкоголь або курить тютюн. Він також
дотримувався строгої веганської дієти протягом 10 років. Всі його страви з високим вмістом листяних
зелених овочів. Лабораторні тести показують гематокрит 35% з нормальною шириною розподілу РБК.
Результати мазка периферичної крові показані на зображенні.

Яке з наступного найістотніше визначатиме ймовірну причину стомлення пацієнта?

A. Народний рівень

B. Рівень гомоцистеїну

C.*Рівень метилмалоновой кислоти

D. Рівні сироваткового гастрину

E. Рівень вітаміну В12 у сечі


37.
37. A 71-year-old man comes to the office because of worsening fatigue. He has noticed difficulty climbing
stairs and shortness ob breath after a casual walk around his neigborhood. His symptoms have progressed
over the past month. He denies orthopnea or hematuria. Past medical history is notable for a mechanical
heart valve placement after cardial auscultation revealed a blowing diastolic murmur 2 years ago. Vitals
sings are within normal limits. Physical examination reveals conjunctival pallor; no jugular venous
distension is noted. Lunds are clear to auscultationand percussion bilaterally. Cardiac examination reveals a
regular rate and rhythm. There is no clubbing of the fingers, cyanosis, or edema. Laborotory result show:

Hemoglobin: 10,9g/dL

Haptoglobin: 19mg/L

Hematocrit: 34%

Prothrombin time: 12 sec

Partial thromboplastin time: 32 sec

Reticulocyte count: 6,2%

Bilirubin total: 3,4 mg/dL

Bilirubin direct: 0,2 mg/dL

Platelet: 340,000/mm3

Lactate dehydrogenase: 230 U/L

A peripheral blood smear is shown.

Which of the following is the most likely cause of this patient's condition?

A. Beta chain hemoglobinopathy

B. Consumptive coagulopathy

C. *Microangiopathic hemolytic anemia

D. Paroxymal nocturnal Hemoglobinuria

E. RBC membrane defect

37. 71-річний чоловік приходить в офіс через погіршення втоми. Він помітив утруднення підйому по
сходах і короткочасного дихання після випадкової прогулянки по сусідству. Його симптоми
прогресували за останній місяць. Він заперечує ортопное або гематурію. Попередня історія хвороби
була помітна для розміщення механічних клапанів серця після аускультації на серці, що виявило
дмеастолічний шум 2 роки тому. Вітальні співи знаходяться в межах норми. Фізичне обстеження
виявляє кон'юнктивальну блідість; не зазначено розтягнення яремної вени. Лундс зрозумілий для
аускультації і перкусії двосторонньо. Кардіологічне дослідження виявляє регулярну швидкість і ритм.
Ніяких клубочків пальців, ціанозу або набряку немає. Показувати результати роботи:

Гемоглобін: 10,9 г / дл

Гаптоглобін: 19 мг / л

Гематокрит: 34%

Протромбіновий час: 12 сек

Час часткового тромбопластину: 32 сек

Кількість ретикулоцитів: 6,2%

Сума білірубіну: 3,4 мг / дл

Білірубін прямий: 0,2 мг / дл

Тромбоцити: 340,000 / мм3

Лактатдегідрогеназа: 230 Од / л

Показано мазок периферичної крові.

Яка з наведених причин є найбільш імовірною причиною стану цього пацієнта?

A. Бета-ланцюг гемоглобінопатії

B. Споживча коагулопатія

C. *Мікроангіопатична гемолітична анемія

D. Пароксимальна нічна гемоглобінурія

E. Дефект мембрани RBC


38.

38.A 73-year-old woman is brought to the emergency department after passing out on the subway. The
patient states that she is constantly fatigued and that she has had months of severe belly pain that worsens
with eating. She began to pass smelly black stools 3 dayago. Blood pressure is 95/52 mm Hg and pulse is
102/min. Physical examination reveals pale conjunctivae and epigastric tenderness to palpation. Which of
the following is the most Likely diagnosis?

A. Anemia of chronic disease secondary to peptic ulcer disease

B. Hemolytic anemia secondary to formation of autoreactive antibodies

C.* Iron deficiency anemia secondary to blood loss

D. Iron deficiency anemia secondary to pernicious anemia

E. Megaloblastic anemia secondary to blood loss

38.73-річну жінку доставляють до відділення невідкладної допомоги після того, як вона вийшла на
метро. Пацієнт стверджує, що вона постійно втомилася і що у неї місяці важкого болю в животі, що
погіршується при їжі. Вона почала пропускати смердючі чорні стільці 3 дня. Артеріальний тиск
становить 95/52 мм рт.ст., а пульс 102 / хв. Фізичне обстеження виявляє бліду кон'юнктиву і
епігастральну болючість при пальпації. Який з перелічених варіантів найбільш вірогідний діагноз?

A. Анемія хронічного захворювання, вторинна до виразкової хвороби

Б. Гемолітична анемія вторинна до утворення аутореактивних антитіл

C.*Залізодефіцитна анемія вторинна в крові

D. Залізодефіцитна анемія, вторинна до перніозної анемії

E. Мегалобластная анемія вторинна до крововтрати


39.
39.An 8-year-old boy is brought to the emergency department because of worsening abdominal pain,
womiting, and bloody diarrhea. His mother says that the symptoms began 6 days after a friend's birthday
party where there was a food truck selling burgers. Vital sings are within normal limits. Conjunctival pallor
is noted. There is mild tenderness to palpation of the abdomen, but no rebound tenderness, hepatomegaly,
or costovertebral angle tenderness. Laboratory studies show the following:

Hemoglobin:8g/dL

Haptoglobin: 20mg/dL

Platelet count: 42,000

Reticulocyte: 12%

Lactate dehydrogenase: 640U/L

Prothrombin time and activated partial thomboplastin time are within normal ranges. A peripheral blood
smear is shown.

Which of the following is the most likely cause of this patient's condition?

A. Disseminated intravascular coagulopathy

B. *Hemolytic uremic syndrome

C. Immune thrombocytopenia

D. Trombotic trombocytopenic purpura

E. Viral Gastroenteritis

39.8-річного хлопчика доставляють до відділення невідкладної допомоги через погіршення болю в


животі, жіночої хвороби та кривавої діареї. Його мати каже, що симптоми почалися через 6 днів після
святкування дня народження друга, де існувала вантажівка з продуктами харчування, яка продавала
гамбургери. Вітальні співи знаходяться в межах норми. Відзначається кон'юнктивальна блідість.
Спостерігається легка болючість при пальпації живота, але немає відскоку болючості, гепатомегалії
або болючості кута нахилу. Лабораторні дослідження показують наступне:

Гемоглобін: 8 г / дл

Гаптоглобін: 20 мг / дл

Кількість тромбоцитів: 42,000

Ретикулоцит: 12%

Лактатдегідрогеназа: 640U / л

Протромбіновий час і активоване часткове час тромбопластину знаходяться в межах норми. Показано
мазок периферичної крові.

Яка з наведених причин є найбільш імовірною причиною стану цього пацієнта?

A. Дисемінований внутрішньосудинний коагулопатія

B. *Гемолітичний уремічний синдром


C. Імунна тромбоцитопенія

D. Тромботична тромбоцитопенічна пурпура

E. Вірусний гастроентерит
40.

40.A 20-year-old college student presents with complaints of severe fatigue, lethargy and painful fingers
and toes in association with changes in the season. She reports that she recently had mono, but her
symptoms have resolved. Physical examination reveals scleral icterus, cervical lymphadenopathy,and
splenomegaly. The tips of the patient's fingers are purple. Laboratory testing shows a decreased
hemoglobin level , an increased reticulocyte count, and a positive result on heterophile testing/

Which of the following is the most likely diagnosis?

A. Aplastic anemia

B. Disseminated intravascular coagulation

C. IgG-mediated hemolytic anemia

D.*IgM-mediated hemolytic anemia

E. Paroxysmal nocturnal hemoglobinuria

40.20-річний студент коледжу подає скарги на сильну втома, млявість і болісні пальці рук і ніг у зв'язку
зі змінами в сезоні. Вона повідомляє, що нещодавно мала моно, але її симптоми вирішилися. Фізичне
обстеження виявляє склеральну жовтяницю, шийну лімфаденопатію і спленомегалію. Верхівки
пальців пацієнта фіолетові. Лабораторні дослідження показують зниження рівня гемоглобіну,
збільшення кількості ретикулоцитів і позитивного результату на гетерофільних тестах /

Який з перелічених варіантів є найбільш вірогідним діагнозом?

A. Апластична анемія

B. Розсіяна внутрішньосудинна коагуляція

C. IgG-опосередкована гемолітична анемія

D.* IgM-опосередкована гемолітична анемія

E. Пароксизмальна нічна гемоглобінурія


41. A 61-year-old Japanese man with a 70-pack-year smoking history and a
chronic cough presents with complaints of night sweats and mild weight loss over the
past several months. He is worried he may be having a reaction to his daughter's new
pet. On physical examination, he has a single swollen lymph node, which is
nontender, nonmobile, hard, and rubbery.

Biopsy of the lymph node is shown.


Which is following is associated with this man's most likely diagnosis?

A. A double-stranded DNA virus

B. A granulomatous reaction to bacterium

C. An idiopathic granulomatous reaction

D. Human T-cell lymphotropic virus type 1

E. The number-one cause of cancer death in the United States

41. 61-річний японець з історією куріння в 70 пачко-років і хронічним кашлем


скаржиться на нічний піт і помірну втрату ваги за останні кілька місяців. Він
турбується, що, можливо, у нього реакція на нову домашню тварину доньки.
При медичному огляді у нього виявлено один припухлий лімфатичний вузол,
який не викликає дискомфорту при пальпації, нерухомий, твердий і
еластичний.

Біопсія лімфатичного вузла показана.

Що з наступного найімовірніше пов’язано з діагнозом чоловіка?

А. ДНК-вірус

В. Гранулематозна реакція на бактерію

С. Ідіопатична гранулематозна реакція


D. Т-лімфотропний вірус людини першого типу

E. Головна причина смерті від раку в Сполучених Штатах


42.

42. A 34-year-old woman comes to the hospital to deliver a term infant. Labor is
complicated by an amniotic fluid embolism, and subsequent blood tests show an
increased level of fibrin split products. The next day, the mother abruptly develops
gross hematuria and flank pain and produces 100 mL of urine over the course of 24
hours. That night, the resident on call notes a scratching sound on cardiac
auscultation.

Which of the following is the most appropriate immediate treatment?

A. Aggressive fluid support

B. Biopsy to evaluate for malignancy

C. Broad-spectrum antibiotics

D. Conservative management

E. *Dialysis

42. 34-річна жінка приїхала в лікарню, щоб народити доношену дитину. Пологи
ускладнюються емболією навколоплідними водами, і наступні аналізи крові
показують підвищений рівень продуктів розщеплення фібрину. На наступний
день у матері раптово розвивається сильна гематурія, біль у боці і протягом 24
годин виділяється 100 мл сечі. Вночі черговий ординатор відзначає дряпаючий
звук при аускультації серця.

Що з наступного є найбільш відповідним негайним лікуванням?


А. Інфузійна терапія

В. Біопсія для оцінки злоякісності

С. Антибіотики широкого спектра дії

D. Консервативне лікування

E. *Діаліз
43.

43.A 25-year man presents to the emergency department due to abdominal pain and
swelling. He denies having a bowel movement for the past week. An abdominal CT
revealed a significant mass. Biopsy of the mass reveals sheets of highly mitotic,
basophilic lymphocytes surrounding clear zones of macrophages.

Which of the following diseases is associated with this histologic picture?

А. Acute lymphoblastic lymphoma

В. *Burkitt lymphoma

C. Follicular lymphoma

D. Multiple myeloma

E. Polycythemia vera

43. 25-річний чоловік надходить до відділення невідкладної допомоги з


причини абдомінального болю і набряку. Він заперечує наявність випорожнень
минулого тижня. КТ черевної порожнини виявила значне ущільнення. Біопсія
ущільнення показує шари високомітотичних, базофільних лімфоцитів, що
оточують чіткі зони макрофагів.

Яке з наступних захворювань пов'язано з цією гістологічною картиною?


А. Гостра лімфобластна лімфома

В. *Лімфома Беркітта

С. Фолікулярна лімфома

D. Множинна мієлома

E. Справжня поліцитемія
44.

44.An 18-year-old man presents to the emergency department with a nosebleed that
started 1 hour ago. He has no significant medical history but says that his gums tend
to bleed quite a bit after he brushes his teeth.

Laboratory tests reveal increased bleeding time and increased partial thromboplastin
time. His mother arrives in the emergency department, and when asked about family
history, says that she has been anemic in the past and always has «a lot of bleeding»
during her menstrual period.

Which of the following is the most likely diagnosis?

A. Bernard-Soulier syndrome

B. Glanzmann disease

C. Hemophilia A

D. Vitamin K deficiency

E. *Von Willebrand disease

44.18-річний хлопець надходить у відділення невідкладної допомоги з


кровотечею з носа, яка почалася годину тому. У нього немає значного
медичного анамнезу, але він каже, що його ясна часто кровоточать після того,
як він чистить зуби.
Лабораторні тести вказують на підвищений час згортання крові і збільшений
частковий тромбопластиновий час. Його мати надходить у відділення
невідкладної допомоги, і коли її питають про сімейний анамнез, вона каже, що
в минулому у неї була анемія і під час менструацій у неї завжди було «багато
кровотеч».

Який з наступних діагнозів є найбільш імовірним?

А. Синдром Бернара-Сульє

В. Хвороба Гланцмана

С. Гемофілія А

D. Дефіцит вітаміну К

E. *Хвороба Віллебранда
45.

45.A 56-year-old man visits his primary care physician at the insistence of his wife.
She is concerned that for the last 4 days, his urine has been “very dark” when he
urinates in the morning, even though he constantly hydrates himself throughout the
day. There is no pain associated with urination. His past medical history is significant
for recurrent leg swelling and pain, for which he has been on warfarin and is now
pain free.

Laboratory studies show:

Hemoglobin: 8.2 g/dL


MCV: 82 fl

Platelets: 134,500/mm3

Flow cytometry reveals a population of cells that are negative for CD55 and C059.

Which of the following laboratory abnormalities is most likely in this patient?

A. Abnormal activated protein C resistance test

B. Elevated branched-chain ketoacids

C. *Elevated urine hemosiderin

D. Elevated urine homogentisic acid

E. IgG, JgM, and C3 deposition along glomerular basement membrane

45. 56-річний чоловік відвідує свого лікаря за наполяганням дружини. Вона


стурбована тим, що протягом останніх 4 днів його сеча була «дуже темною»,
коли він мочився вранці, хоча він постійно пив воду протягом дня. Болю,
пов'язаного з сечовипусканням, немає. Його минула історія хвороби є
важливою для періодичних набряків і болю ніг, через що він приймав
варфарин. Зараз біль відсутній.

Лабораторні дослідження показують:

Гемоглобін: 8,2 г/дл

MCV (середній об’єм еритроцитів): 82 фл

Тромбоцити: 134,500/мм3

Проточна цитометрія виявляє популяцію клітин, які є негативними для CD55 і


C059.

Яка з наведених нижче лабораторних аномалій є найбільш імовірна у даного


пацієнта?

A. Тест на аномальний активований протеїн С

B. Підвищені розгалужені ланцюги кетокислот

C. *Підвищений гемосидерин сечі

D. Підвищена сечова гомогентизова кислота


E. Осадження IgG, ІgM і C3 вздовж клубочкової базальної мембрани
46.

46. A 45-year-old patient presents with fatigue, 10 Іb unintentional weight loss over
the past 2 months, and a hard neck mass, which seems to adhere to adjacent tissue.
The physician orders a biopsy and diagnoses him with lymphoma. Genetic analysis of
the tumor cells shows a translocation from chromosome 18 to chromosome 14.

What is the role of the protein that is overexpressed as a result of this chromosomal
rearrangement?

A. Agonist of apoptotic cell death

B. *Antagonist of apoptotic cell death

C. Cell cycle inhibitor in response to the detection of a DNA error

D. Constitutive activation of RAS

E. Inactivation of p53

F. Upregulation of the cell cycle

46. У 45-річного пацієнта спостерігається втомлюваність, ненавмисна втрата


ваги протягом останніх 2 місяців і велике утворення на шиї, що, здається,
приєднується до сусідньої тканини. Лікар замовляє біопсію і діагностує
лімфому. Генетичний аналіз пухлинних клітин показує транслокацію з 18-ї
хромосоми на 14-у хромосому. У чому полягає роль білка, що
оверекспресується внаслідок цієї реорганізації?
А. Агоніст апоптотичної загибелі клітин

B. *Антагоніст апоптотичної загибелі клітин

C. Інгібітор клітинного циклу у відповідь на виявлення помилки ДНК

D. Конститутивна активація РАC

E. Інактивація p53

F. Регулювання клітинного циклу


47.
47.A 76-year-old woman presents to the emergency department with a 3-day history
of constipation. She reports fatigue and changes in bowel habits over the period of a
year, including dark stools. On physical examination, the patient's abdomen is
distended. Laboratory studies show:

WBC count: 10,200/mm3

Hemoglobin: 8.8 g/dL

Hematocrit: 26,5%

Platelet count: 231,000/mm3

A barium enema CT is performed and is shown.

Which of the following features will likely be present in a peripheral blood smear
from this patient?

A. Hypersegmented neutrophils

B. *Microcytic red cells

C. Normochromic red blood cells

D. Pappenheimer bodies

E. Schistocytes

F. Spherocytes

47.76-річна жінка надходить до відділення невідкладної допомоги з 3-денною


історією запору. Вона повідомляє про стомлюваність і зміни в звичках
кишечника протягом року, включаючи темні випорожнення. При фізичному
обстеженні живіт пацієнтки розтягується. Лабораторні дослідження показують:

Кількість лейкоцитів: 10,200/мм3


Гемоглобін: 8,8 г/дл

Гематокрит: 26,5%

Кількість тромбоцитів: 231,000/мм3

КТ з барієвою клізмою проведено та показано.

Яка з наступних особливостей, ймовірно, буде присутня в мазку периферичної


крові цього пацієнта?

А. Гіперсегментовані нейтрофіли

B. *Мікроцитарні червоні клітини

C. Нормохромні еритроцити

D. Тіла Паппенгаймера

E. Шистоцити

F. Сфероцити
48.

48. An 8-month-old boy is brought into the emergency department by his mother due
to lethargy, poor feeding, and abdominal swelling. The mother notes that his
symptoms have become progressively worse over the past 2 months. She also states
that delivery was full term without any complications. Review of symptoms was
negative. Physical examination reveals growth retardation, hepatosplenomegaly, and
jaundice. CBC findings include the following:
Hematocrit: 27%

Platelet count: 250,000/mm3

Leukocytes: 9,300/mm3

Bleeding time: normal

INR: 1.1

Reticulocyte count: 10%

The physician orders hemoglobin (Hb) electrophoresis, which shows marked


elevation of HbF, increased HbA2, and absence of HbA1.

Which of the following diagnoses is most consistent with this patient's findings?

A. Glucose-6-phosphate dehydrogenase deficiency

B. HbH disease

C. RBCs containing Hb Barts

D. a-Thalassemia minor

E. *β -Thalassemia major

F. β -Thalassemia minor

48. Мати 8-місячного хлопчика привозить його у відділення невідкладної


допомоги через млявість, погане годування та набряк живота. Вона зазначає,
що його симптоми поступово погіршувалися протягом останніх 2 місяців. Мати
хлопчика стверджує, що пологи були у повний термін та без ускладнень. Огляд
симптомів був негативним. Фізичне обстеження виявляє затримку росту,
гепатоспленомегалію і жовтяницю. Висновки клінічного аналізу крові
включають наступне:

Гематокрит: 27%

Кількість тромбоцитів: 250,000/мм3

Лейкоцити: 9,300/мм3

Часзгортання крові: нормальний

Протромбіновий час: 1.1


Кількість ретикулоцитів: 10%

Лікар замовляє гемоглобіновий (Hb) електрофорез, який показує помітне


підвищення HbF, збільшення HbA2 і відсутність HbA1.

Який з наступних діагнозів найбільш відповідає результатам цього пацієнта?

A. Дефіцит глюкозо-6-фосфатдегідрогенази

B. Хвороба HbH

C. RBCs, що містять Hb Barts

D. a-Thalassemia minor

Е. *β -Thalassemia major

F. β -Thalassemia minor
49.

49. A 2-year-old girl is brought to the emergency department because of fevers that
have been occurring daily over the past 3 weeks. Physical examination reveals a 1.2-
kg (2.6-lb) weight loss since her last doctor's visit 1 month earlier. She also has
pallor, hepatomegaly, and splenomegaly. Laboratory tests are shown bellow. She is
admitted to the hospital and undergoes bone marrow aspiration. Results are shown in
the image.

White blood cell: 1.2 x 103/pL

Hemoglobin: 7.4 g/dl

Hematocrit: 23%

Platelets: 109x 103/pL

ALP: 113IU

AST: 871U

ALT: 112 IU

GGT: 51 IU

Ferritin level of 11,000 pg/L.

Which of the following is the most likely diagnosis?

A. Autoimunne lymphoproliferative syndrome

B. Ewing sarcoma

C. *Hemophagocytic lymphohistiocytosis

D. Multiple myeloma

E. Neuroblastoma

F. Wilms tumor

49. 2-річну дівчинку привозять до відділення невідкладної допомоги через


лихоманку, яка виникає щодня протягом останніх 3 тижнів. Фізичне
обстеження виявляє втрату ваги на 1,2 кг після останнього візиту лікаря за
місяць раніше. Вона також має блідість, гепатомегалію і спленомегалію.
Лабораторні тести показані нижче. Дівчинка госпіталізована і проходить
аспірацію кісткового мозку. Результати показані на зображенні.

Лейкоцити: 1,2 x 103/ μL

Гемоглобін: 7,4 г/дл

Гематокрит: 23%

Тромбоцити: 109х103/ μL

АЛП: 113 IU

АСТ: 87 ІU

АЛТ: 112 МО

ГГТ: 51 МО

Рівень феритину 11000 μг/л.

Який з перелічених варіантів є найбільш вірогідним діагнозом?

A. Аутоімунний лімфопроліферативний синдром

B. Саркома Юінга

C. *Гемофагоцитарний лімфогістіоцитоз

D. Множинна мієлома

E. Нейробластома

D. Пухлина Вільмса

50.
50.A 59-year-old woman is admitted to the hospital because of a brief episode of
right-sided paralysis. Medical history is significant for an 8-week history of bleeding
from the gums and the nose, and throbbing, burning sensations in the hands and feet.
Physical examination reveals mild left upper quadrant pain; palpation demonstrates
an enlarged spleen. Blood test show:

Hemoglobin level: 15 g/dL

Hematocrit: 45%

Platelet count: 900,000/mm3

Erythrocyte sedimentation rate: 12 mm/h

C-reactive protein: 1.2 mg/L (normal range 1.0-3.0 mg/L)

WBC count: 8,400/mm3

Based on the patient's symptoms and abnormal laboratory results, the physician
prescribes treatment with an antineoplastic agent.

Which is following is the most likely diagnosis?

A. *Essential thrombocythemia

B. Immune thrombocytopenia
C. Polycythemia vera

D. Sickle cell disease

E. Thrombotic thrombocytopenic purpura

50. 59-річна жінка потрапила в лікарню через короткий епізод


правостороннього паралічу. Анамнез хвороби є важливим для 8-тижневої
історії кровотечі з ясен і носа, а також пульсуючих, пекучих відчуттів у руках і
ногах. Фізичне обстеження виявляє незначний біль лівого верхнього квадранта;
пальпація демонструє збільшену селезінку. Аналіз крові:

Рівень гемоглобіну: 15 г/дл

Гематокрит: 45%

Кількість тромбоцитів: 900000/мм3

Швидкість осідання еритроцитів: 12 мм/год

С-реактивний білок: 1,2 мг/л (нормальний діапазон 1,0-3,0 мг / л)

Кількість лейкоцитів: 8,400 / мм3

На підставі симптомів хвороби та аномальних лабораторних результатів, лікар


призначає лікування протипухлинним засобом.

Який наступний діагноз є найбільш імовірним?

A. *Есенціальна тромбоцитемія

B. Імунна тромбоцитопенія

C. Справжня поліцитемія

D. Серповидноклітинна анемія

E. Тромботична тромбоцитопенічна пурпура


51.

51. A 32-year-old woman reports low-grade-fevers, Night sweats, and feeling tired for the past 2 month.
She has nontender cervical and supraclavicular lymphadenopathy. The physician performs a cervical lymph
node biopsy; the findings are shown in the image. This patient’s diagnosis is associated with which of the
following?

A. Cells that are positive for CD15 and CD30

B. *Endemic presence in African children


C. Lytic lesions on a skeletal survey

D. Presence of CMV antibodies

E. t (14;18) translocation

32-річна жінка скаржиться на низьку температуру, нічну пітливість і відчуття втоми протягом останніх
2 місяців . Вона має некротичну цервікальну та надключичну лімфаденопатію . Лікар провів біопсію
лімфатичних вузлів шийки матки; результати показані на зображенні. Діагноз цього пацієнта
пов'язаний з наступним:

A. Клітини, які є позитивними для CD15 і CD30

B. *Ендемічна присутність у африканських дітей

C. Літичні ураження на скелетному обстеженні

D. Наявність ЦМВ-антитіл

E. t (14; 18) транслокації


52.

52.A 69-year-old woman presents to her primary care physician with a 6-month history of increasing
fatigue, weakness and anorexia. Her daughter visited recently and expressed concern that the patient was
taking multiple naps throughout the day , which was unusual for her. Physical examination is remarkable
for pale conjunctiva bilaterally. Her hematocrit is 37%. Her peripheral blood smear is shown. Which of the
following is the most likely mechanism for the findings in the blood smear?
A. Cold agglutinins

B. *Colorectal malignancy

C. Lead poisoning

D. Mutation in the β-globin chain, causing destruction of erythrocytes

E. Nutritional deficiency of folic acid

52. 69-річна жінка скаржиться лікуючому лікарю на збільшення втоми, слабкість і анорексію , які
тривають 6 місяців. Її донька, нещодавно відвідавши ії, висловила стурбованість тим, що мати лягає
придрімати кілька разів удень, що було незвично для неї. При фізичному обстеженні виявлено бліду
двосторонню кон’юнктиву. Гематокрит становить 37%. Показано мазок периферичної крові. Який з
висновків є найбільш імовірним після перегляду мазку крові?

А. Холодні аглютиніни

B. *Колоректальна злоякісність

C. Отруєння свинцем

D. Мутація в β-глобіновому ланцюзі, що викликає руйнування еритроцитів

E. Харчовий дефіцит фолієвої кислоти


53.

53. A 16-year-old boy comes for evaluation because of a 2-week history of weakness, fever, and anorexia.
Temperature is 39.1 oC (102.3 oF). The patient reports swelling around his neck and groin region with
multiple patches of small red dots. A CT scan of the chest is shown below. Selected results from a complete
blood count include :

White blood cells coun: 18,000/mm3

Hemoglobin : 115g/dL

Platelet count: 120,000/mm3

Which of the following best describes the predominant cell type that is likely to be found on a lymph node
biopsy?

A. Pre-B-cells, terminal deoxynucleotidyl transferase positive


B. B cells, CD 15 and CD 30+
C. Memory B cells, tartrate resistant acid phosphatase positive
D. Metamyelocytes, translocation t(9:22)
E. *Pre-T-cells, terminal deoxynucleotidyl transferase positive

53. 16-річний хлопчик приходить на обстеження через 2-тижневу слабкість, лихоманку та анорексію.
Температура становить 39,1 oC (102,3 oF). Пацієнт повідомляє про набряк навколо шиї і пахової області
з декількома плямами маленьких червоних крапок. КТ грудної клітки показана нижче. Вибрані
результати повного аналізу крові включають:

Білі клітини крові: 18000 / мм3

Гемоглобін: 115г / дл

Кількість тромбоцитів: 120,000 / мм3


Яке з наведених нижче тверджень описує переважний тип клітин, який може бути знайдений на
біопсії лімфатичних вузлів?

А. Пре -В-клітини, кінцева дезоксинуклеотидилтрансфераза позитивна

B. В-клітини, CD 15 і CD 30+

В. В-клітини пам’яті , стійкі до тартратної кислої фосфатази

D. Метаміелоцити, транслокація t (9:22)

E. *Пре-Т-клітини, кінцеві дезоксинуклеотидилтрансферази позитивні


54.

54. A 15-year-old African – American girl is brought to the emergency department because of extreme pain
in her right foot and abdomen. On physical examination, her foot and two of her toes are found to be pale
and cold, and she winces with light touch. A computed tomography (CT) scan of the abdomen shows an
abnormally small spleen. She received brisk analgesia and fluid resuscitation and recovered a few days
later. On discharge, she was prescribed a medication to reduce the number of hospitalization. What is the
main clinical benefit of the treatment prescribed at discharge?

A. Decrease in viscosity of blood to improve blood flow


B. *Increase in a particular hemoglobin type in circulation
C. Inhibition of bacterial mitosis
D. Replacement of diseased blood cells with fresh blood cells
E. Stabilization of hemoglobin in the R state to maintain oxygenation

54.15-річна афро-американська дівчина була доставлена до відділення швидкої допомоги через


сильний біль у правій нозі та животі. При фізичному огляді, її нога й два пальці були блідими й
холодними, а вона здригалась від легкого дотику. Комп'ютерна томографія (КТ) живота показує
аномально малу селезінку. Дівчина отримала знеболювання й інфузійну терапію та одужала через
кілька днів. При виписці пацієнтці призначили ліки, щоб зменшити число госпіталізацій. Який
основний клінічний ефект від лікування, що призначається при виписці?

A. Зниження в'язкості крові для поліпшення кровотоку


B. *Збільшення певного типу гемоглобіну в кровообігу
C. Інгібування бактеріального мітозу
D. Заміна хворих клітин крові свіжими клітинами
E. Стабілізація гемоглобіну в R-стані для підтримки оксигенації
55.
55. A 16-year-old girl visits her physician for a routine check-up. She reports increased tiredness while at
school for the past 3 weeks but otherwise feels fine. On physical examination, her temperature is 38 oC
(100.4 oF), blood pressure is 112/78 mm Hg, pulse is 88/min, and respiratory rate 16/min. She has mild
pharyngitis on inspection of the oropharynx and tender axillary lymphadenopathy is also noted. Her spleen
is palpable, and her liver feels mildly enlarged on abdominal examination. A complete blood count shows
the following:

Hemoglobin: 13g/dL

Hematocrit: 41%

Mean corpuscular volume: 88 fL

Platelet count: 300,000/mm3

WBS count: 9000/mm3

A rapid streptococcus test is negative. However, reactive heterophile antibodies that agglutinate horse red
blood cells are found in the patient’s serum. Peripheral blood smear is performed and an image from the
smear is shown.

That virus that causes the infection described above is associated with what other disease?

A. Acute glomerulonephritis
B. Cervical carcinoma
C. MALT lymphoma
D. *Nasopharyngeal carcinoma
E. Rheumatic heart disease

55. 16-річна дівчинка відвідала свого лікаря для планової перевірки. Вона скаржиться на збільшення
втомлюваності, перебуваючи в школі протягом останніх 3 тижнів, але в іншому почувається добре.
При медичному огляді її температура становила 38 oC (100,4 oF), артеріальний тиск 112/78 мм рт.ст.,
пульс 88 / хв, частота дихання 16 / хв. При огляді ротоглотки виявлено легкий фарингіт і помірну
пахвову лімфаденопатію. Її селезінка пальпується, печінка помірно збільшена при абдомінальному
дослідженні. Повний аналіз крові показує наступне:

Гемоглобін: 13 г / дл

Гематокрит: 41%

Середній корпускулярний об'єм: 88 фл

Кількість тромбоцитів: 300000 / мм3


Кількість WBS: 9000 / мм3

Швидкий тест на стрептокок негативний. Проте в сироватці пацієнта виявляються реактивні


гетерофільні антитіла, які агглютинують еритроцити коня. Здійснено мазок периферичної крові:

Вірус, який викликає описану вище інфекцію, пов'язаний з таким захворюванням?

A. Гострий гломерулонефрит

B. Рак шийки матки

C. MALT-лімфома

D. *Назофарингеальна карцинома

E. Ревматична хвороба серця


56.

56. A Japanese male present to the emergency department due to the constipation and erectile
dysfunction. The patient admits to having frequency used IV heroin in the past as well as having multiple
sexual partners. Physical exam reveals a palpable spleen in the right lateral decubitus position and a liver
span of 20 cm. The patient also has diffuse nodular cutaneous lesions on his upper and lower extremities.
Laboratory studies reveal hypercalcemia. A peripheral blood smear of a T cell is shown.
Which of the following is implicated in the pathogenesis of this man’s disease?

A. Epstein-Barr virus

B. HIV

C. Hepatitis C virus

D. *Human T-cell leukemia virus

E. Human herpesvirus 8

56. Японський чоловік доставлений у відділенні невідкладної допомоги через запор та еректильну
дисфункцію. Пацієнт повідомляє, що в минулому часто приймав гепарин IV, а також мав декілька
статевих партнерів. При фізичному огляді виявлено селезінку, що пальпується; в правому боковому
положенні провисання і прогин печінки становить 20 см. Пацієнт також має дифузні вузлові
ураження шкіри на верхніх і нижніх кінцівках. Лабораторні дослідження виявляють гіперкальціємію.
Показано мазок периферичної крові Т-клітин.

Що викликає розвиток цієї хвороби?

A. Вірус Епштейна-Барра

B. ВІЛ

C. Вірус гепатиту С

D. *Вірус Т-клітинного лейкемії людини

E. Вірус герпесу 8 людини


57.

57. A 24-yaer-old previously healthy woman in the final stages of labor suddenly complains of shivering and
extreme chills. She subsequently becomes short of breath and begins to vomit. Further IV access is sought
and the patient oozes blood around her IV sites. Her blood pressure is taken and is 77/51 mm Hg. Minutes
later, small ecchymoses appear on her legs. The baby and placenta are delivered, but the obstetrician is
unable to control the patient’s vaginal bleeding.

Choise Platel count Bleeding time Prothrombin time Partial


thromboplastin
time
A normal normal normal ↑
B normal normal ↑ normal
C normal ↑ normal normal
D normal ↑ normal ↑
E ↓ ↑ normal normal
F ↓ ↑ ↑ ↑
Referring to the table, which is this woman’s most likely coagulation profile?

A. A
B. B
C. C
D. D
E. E
F. *F

57.24-річна жінка, яка раніше була здоровою, на кінцевих стадіях пологів раптово скаржиться на
тремтіння і сильні озноби. Згодом вона задихається і починає блювати. Подальший доступ до ІВ
шукається , і пацієнтку просочує кров навколо її IВ ділянок. Кров'яний тиск становить 77/51 мм рт.
Через кілька хвилин на її ногах з'являються невеликі ехімози. Дитина і плацента народжуються, але
акушер не в змозі контролювати вагінальні кровотечі пацієнта.

Вибір Кількість Час кровотечі ПротромбіновийЧастковий


тромбоцитів час тромбопластиновий
час
A норма норма норма ↑
B норма норма ↑ норма
C норма ↑ норма норма
D норма ↑ норма ↑
E ↓ ↑ норма норма
F ↓ ↑ ↑ ↑
Посилаючись на таблицю, що є найбільш імовірним профілем згортання цієї жінки?

A. A

B. B

C. С

D. D

Е. E

F. *F
58.

58. A 16-year-old boy with a history of nosebleeds are frequent bruising comes to his pediatrician because
he recently began passing blood-tinged stool. Physical examination reveals mild gingival bleeding.
Laboratory studies show:

Platelets: 250 × 103 / µL

Prothrombin time: 13 sec

Partial thromboplastin time: 49 sec

Bleeding time: 15 min

Which of the following is the function of the protein most likely mutated in this patient?

A. *Acts as a ligand in platelet adhesion


B. Acts as a receptor for fibrinogen
C. Acts as a receptor for von Willebrand factor
D. Cleaves large von Willebrand factor multimers to proper size
E. Inactivates factors V and VIII

58. 16-річний хлопчик з носовими кровотечами й нерідкими синцями в анамнезі звернувся до свого
педіатра, тому що він недавно помітив кров'яні випорожнення. Фізичне обстеження показує помірну
кровотечу ясен. Лабораторні дослідження показують:

Тромбоцити: 250 × 103 / мкл

Протромбіновий час: 13 сек

Частковий тромбопластиновий час: 49 сек

Час кровотечі: 15 хв

Яка з наведених нижче функцій білка, швидше за все, мутувала у цього пацієнта?
А. *Діє як ліганд в адгезії тромбоцитів

B. Діє як рецептор фібриногену

C. Діє як рецептор фактора Віллебранда

Д. Розколює великий мультимер фактора Віллебранда до відповідного розміру

E. Інактивує фактори V і VIII


59.

59. A 40-year-old woman is admitted to the hospital because of bloody, painful urination and extreme
tenderness of the lower back. The patient also noted fever, chills and malaise for the past 2 days. Her
temperature is 98.4 oF (36.9 oC) , pulse is 110 / min, and respiratory rate is 26 / min. She is intubated and
antibiotic treatment is started. The following day, the patient has multiple petechiae and ecchymoses on
the trunk and legs. Blood is also noted around her mouth and nose. A peripheral blood smear will most
likely show which of the following?

A. Acanthocyte
B. Only normal RBCs
C. *Schistocytes
D. Spherocytes
E. Target cell

59. 40-річну жінку госпіталізують у лікарню через криваві, болісні сечовипускання та крайню
болючість нижньої частини спини. Пацієнтка також відзначає лихоманку, озноб і нездужання
протягом останніх 2 днів. Її температура становить 98,4 ° С (36,9 ° С), пульс 110 / хв, а частота дихання
26 / хв. Її інтубують і починають лікування антибіотиками. На наступний день у хворої є кілька петехій і
екхімоз на тулубі і ногах. Кров також відзначається навколо її рота і носа. Мазок периферичної крові,
швидше за все, покаже:

А. Акантоцити

B. Тільки звичайні еритроцити

C. *Шистоцити

D. Сфероцити

E. Цільова клітина
60.

60. A 19-year-old man present with a 1-week history of fever, weakness, and fatigue. Physical examination
shows pale conjunctivae and petechiae spread diffusely across his trunk. A chest X-ray is shown. A biopsy
reveals cells with light blue, scanty, and nongranular cytoplasm when stained. Results of his CBC are as
follows:

Hemoglobin: 7g/dL
Hematocrit: 21%

Reticulocyte count: 0.1% (normal 0.5% - 1.5%)

Platelet count: 50,000/mm3

Leukocytes: 17,300/mm3

Neutrophils: 44%

Eosinophils: 2%

Lymphocytes: 46%

Monocytes: 6%

Which of the following is the most likely diagnosis?

A. B-cell acute lymphoblastic lymphoma


B. Chronic lymphocytic leukemia
C. Follicular lymphoma
D. Hodgkin disease
E. *T-cell acute lymphoblastic lymphoma

60. 19-річного чоловік звернувся з тижневою лихоманкою, слабкістю та втомою. Фізичне обстеження
показує бліді кон'юнктиви, а петехії поширюються дифузно поперек його тулуба. Показано рентген
грудної клітки. Біопсія виявляє клітини з світло-блакитною, мало- і негранулярной цитоплазмою при
фарбуванні. Результати його CBC є такими:

Гемоглобін: 7 г / дл

Гематокрит: 21%

Кількість ретикулоцитів: 0,1% (нормальне значення 0,5% - 1.5%)

Кількість тромбоцитів: 50,000 / мм3

Лейкоцити: 17,300 / мм3

Нейтрофіли: 44%
Еозинофіли: 2%

Лімфоцити: 46%

Моноцити: 6%

Який з перелічених варіантів є найбільш вірогідним діагнозом?

A. B-клітинна гостра лімфобластна лімфома

B. Хронічний лімфолейкоз

C. Фолікулярна лімфома

D. Хвороба Ходжкіна

E. *Т-клітинна гостра лімфобластна лімфом


61.

61.A 52-year-old man presents to a physician complaining of a 4 month history of fatigue, a sore tongue,
and tingling in his fingers and toes. Findings on physical examination are unremarkable. Medical records
from the patient`s former physician indicate that the patient has a history of pernicious anemia. His serum
vitamin B 12 level is low. Which of the following findings confirms a diagnosis of pernicious anemia in this
patient?

A. Low levels of urinary vitamin B 12 in stage 1 and stage 2 of a Schilling test


B. *Low levels of urinary vitamin B 12 in stage 1 of a Schilling test, normal levels of urinary vitamin B
12 in stage 2 of a Schilling test
C. Normal levels of urinary vitamin B 12 in stage 1 and stage 2 of a Schilling test
D. Normal levels of urinary vitamin B 12 in stage 1 of a Schilling test, low levels of urinary vitamin B 12
in stage 2 of a Schilling test

61.Чоловік скаржиться лікареві, що в нього 4-місячна назад з’явилось втома, хворий язик, і
відчуваючи дзвін у вухах в його пальцях і пальцях на нозі. Виявлення на медичному огляді нечудові.
Медичні записи пацієнта колишнього лікаря вказують, що пацієнт має історію згубної анемії. Його
серум вітамінний В12 рівень низький. Яке з наступних виявлень підтверджує діагноз згубної анемії у
цього пацієнта?

A. Низькі рівні сечового вітаміну в стадії 1 і стадія 2 з випробування Шилінга.

B. *Низькі рівні сечового вітаміну в стадії 1 з випробування Шилінга, нормальні рівні сечового в стадії
2 з випробування Шилінга

C. рівні нормального Стану сечового вітаміну в стадії 1 і стадія 2 з випробування Шилінга.

D. рівні нормального Стану сечового вітаміну в стадії 1 з випробування Шилінга, низькі рівні сечового
вітаміну в стадії 2 of випробування Шилінга
62.

62.An 11-year-old African-American boy begins treatment for tuberculosis. Five days later. He is brought to
the emergency department because of severe fatigue, lethargy, and pallor. Laboratory testing reveals:

Hemoglobin: 8 g/dL
Hematocrit: 24%
MCV: 85
Platelet count: 180,000/mm^3
Leukocytes: 7,300/mm^3
Neutrophils: 58%
Eosinophils: 2%
Lymphocytes: 34%
Monocytes: 6%
LDH 250IU/L (normal 56-194)
Reticulocyte count: 4%
A decrease of which of the following is responsible for this patient`s findings?

A. Cobalamin (vitamin B 12)


B. Collagen type IV
C. *Nicotinamide adenine dinucleotide phosphate (NADPH)
D. Pyridoxine (vitamin B6)
E. Thiamine (vitamin B 1)
F. Tyrosine

62.Хлопчик 11 років з Африки почав лікування з приводу туберкульозу. 5 днів згодом, він навідався
до відділення невідкладної допомоги із-за строгої втоми, летаргії, і блідості. Лабораторне
випробування виявляє:
Гемоглобін: 8g/dL
Гематокрит: 24%
Середній об’єм еритороцитів: 85
Кількість тромбоцитів: 180,000/mm^3
Лейкоцитів: 7,300/mm^3
Нейтрофіли: 58%
Еозінофіли: 2 %
Лімфоцити: 34%
Моноцити: 6%
ЛДГ: 250 IU/L (норма 56-194)
Кількість ретікулоцитів: 5%

Зменшення, якого з наступного ку речовин, відповідальне за виявлення стану цього пацієнта?

A. Кобаламін (вітамін B12)


B. Колаген IV
C. *Нікотинамід-аденін-динуклеотид-фосфат (НАДФ)
D. Піродиксин (вітамін В6)
E. Тіамін (вітамін В1)
F. Тірозин
63.

63.

A 15-year-old boy is brought to the emergency department because of a nosebleed that has lasted for the
past hour. The patient and his parents deny any recent trauma before the bleeding. On review of systems,
the patient confirms an episode of rhinorrhea and cough6 weeks ago. He also states that he noticed his
legs and arms have bruised more easily in the past month. The patient also denies using any medications or
illicit drugs and claims that he is not sexually active. The physical examination is notable only for small
nonblanching lesions on the lower legs, as shown in the image. The spleen is not palpable.

Laboratory testing reveals:


Hematocrit: 40%
Platelet count: 8,000/mm^3
Leukocytes: 5,300/mm^3
Neutrophils: 58%
Eosinophils: 2%
Lymphocytes: 34%
Monocytes: 6%
Fibrinogen: 200 mg/dL
INR: 1.1

Would of the following additional laboratory findings would most likely be present in this patient?

A. *Antiplatelet antibodies
B. Decreased megakaryocytes on bone marrow biopsy
C. Increased fibrin split products
D. Schistocytes on peripheral blood smear
E. Vitamin K deficiency

63. Хлопчик 15 років прийшов до відділення невідкладної допомоги через кровотечі з носа, яка
тривала одну минулу годину. Пацієнт і його батьки відхиляють будь-яку недавню травму перед
кровотечею. На огляді систем, пацієнт підтверджує епізод кровотечі з носа і кашля 6 тижнів тому. Він
також заявляє, що він помітив, що його ноги і руки легше забиваються в минулому місяці. Пацієнт
також відхиляє користування будь-якими лікарськими засобами або незаконними наркотиками і
стверджує, що він сексуально не активний. Медичний огляд відомий тільки для маленьких
небліднучих ушкоджень на ногах, як показано в зображенні. Селезінка не пальпується.

Лабораторне випробування виявляє:


Гематокрит: 40%
Кількість тромбоцитів: 8000/mm3
Лейкоцити: 5300/mm3
Нейтрофіли: 58%
Еозінофіли: 2 %
Лімфоцити: 34%
Моноцити: 6%
ЛДГ: 200 IU/L (норма 56-194)
Протромбиновий час: 1.1

Яке з додаткових лабораторних виявлень вірогідніше всього був би присутнім у цього пацієнта?

A. *Антипластинки антитіл
B. Зменшений meгакаріоцитів на біопсії кісткового мозкукістки
C. Була підвищена продукція розколу фібрину
D. Цистоскопія мазка периферичної крові
E. Дефіцит вітаміна К
64.

64. A 45-year- old man presents to his internist with worsening fatigue and myalgias over the past several
months. He described his thinking as clouded, and states that, especially with tasks requiring significant
concentration, he is unable to focus and think logically anymore. He states, `` I just feel so sluggish and
slow. I am not the way I used to be``. He lives alone in a downtown apartment building and works in the
local shipyard. On examination, he appears pale and fatigued. A peripheral blood smear reveals red blood
cells with dark-purple, punctate inclusions. Laboratory tests show:

WBC: 6400/mm^3
RBC: 4.08million/mm^3
Hb: 9.8g/dL
Hct: 29.6%
MCV: 72 mm^3
Platelets: 270,000/mm^3
Which of the following metabolites is most likely elevated in this patient`s blood?
A. *Aminolevulinic acid
B. Glycine
C. Porpohobilinogen
D. Succinyl-coenzyme A
E. Uroporphyrinogen III

64. Чоловік 45 років прийшов до його терапевта зі скаргами на підвищену втому та міалгію за
останні декілька місяців. Він описує свій роздум, як захмарення, і заявляє що це, особливо пов’язано
із завданнями, що вимагають істотної концентрації, він не в змозі фокусуватися і думати логічно. Він
заявляє, ''я тільки почуваю себе таким в'ялим і повільно. Я – не можу використати цей шлях щоб бути
собою. '' Він живе тільки в розташованій діловий частині міста в кімнатній будівлі і працює в місцевій
верфі. На огляді, він виглядав блідим і стомленим. Периферійний кров'яний мазок виявляє червоні
кров'яні осередки з темним пурпуром, плямисті включення. Лабораторні випробування показують:

Лейкоцити: 6400/mm^3
Еритроцити: 4.08 million/mm^3
Гемоглобін: 9.8 g/dL
Гематокрит: 29.6%
Середній об’єм еритроцитів: 72 mm^3
Концентрація гемоглобіну в еритроциті: 30.1%
Тромбоцитів: 270,000/mm^3

З якими наступними метаболітами найбільш пов’язан стан крові пацієнта?

A. *Амінолевулінова кислота
B. Гліцин
C. Порфобіліноген
D. Сукциніл-коензім А
E. Уропорфіріноген 3
65.

65. A 27-year-old white man presents with pain and swelling in his right calf, which started overnight. He
denies any recent travel, long car trips, or surgeries. Notably, he presented with similar calf pain 3 years
earlier and has a brother who had an infraction of his mesenteric vein at the age of 35. On physical
examination, his blood pressure is 135/70 mm Hg, pulse is 80/min, respiratory rate is 14/min, and oxygen
saturation is 99% on room air. Physical examination of his right calf reveals it to be erythematous and
tender, with a circumference 2 cm greater than that of the left calf. Which of the following disorders does
this patient most likely have?

A. Antiphospholipid antibody syndrome


B. *Factor V Leiden mutation
C. Protein S deficiency
D. Prothrombin gene mutation
E. Von Willebrand factor deficiency

65. Чоловік звернувся з болем і припухлістю з його правої частини гомілки, яке було запущене
напередодні увечері . Він відхиляє будь-яку недавню подорож, довгі автомобільні подорожі, або
хірургію. Не виключно, що він мав подібну біль гомілки 3 роками раніше і має брата, який мав
порушення його брижової вени у віці 35. На медичному огляді, його кров'яний тиск 135/70, пульс
80/хв, дихальній показник 14/хв, і киснева насиченість 99% у повітрі кімнати. Медичний огляд його
правої гомілки виявляє це, щоб бути eритроматозний і м'який, з колом 2 см, більші, ніж та ліва
гомілка. Яке з наступних порушень пацієнт вірогідніше всього має?

A. Синдром антитіла Антифосфоліпіду


B. *Фактор 5 мутації Лейдена
C. Дефіцит Білка S
D. Мутація гена протромбіна
E. Дефіцит фактору вон Виллібранда
66.

66. A 65-year-old African-American woman is brought to the emergency department in a combative state
after found wandering aimlessly in a park. After mild sedation is achieved, the patient repeatedly complains
of a ``pain in my head`` and `` not being able to move my left foot properly``. The patient is febrile on
admission, and multiple punctate red lesions are visible on the inside of her mouth. Platelet count is
10,000/mm^3, and serum creatinine level is 6.3 mg/dL. A blood smear is shown in the image. The patient
undergoes plasma exchange for several days and is then discharged.
What is the most likely cause of this patient`s symptoms?

A. Abnormally high blood ammonia levels


B. An immune attack against platelet surface antigens
C. *Antibodies directed the against the ADAMTS13 metalloprotease
D. Erythrocytes that contain denatured hemoglobin
E. Long-standing hypertension

66. Африко-американська жінка 65 років звернулась до відділення невідкладної допомоги у бойовій


державі, будучи знайденою, у той момент, коли вона бродила безцільно в парку. Після того, як її
заспокоїли, пацієнтка повторно скаржиться на ''біль в моїй голові'' і ''не була в змозі перемістити мою
ліву сторону стопи належним чином''. Пацієнтка мала високу температуру, і багаторазові плямисті
червоні ушкодження видимі з внутрішньої сторони її рота. Тромбоцитів складає 10,000/mm^3, і
рівень серум креатиніну є 6.3 mg/dL. Кров'яний мазок показується в зображенні. Пацієнт випробовує
плазмовий обмін впродовж декількох днів і потім звільнений від зобов'язань його робити .

Яка найбільш вірогідна причина симптомів пацієнта:

A. Ненормально високі показники рівня амонію


B. Атака імунної системи на антигени поверхні тромбоцитів
C. *Антитіла спрямовані на ADAMTS13 металопротезу
D. Еритроцити, які містять денатурований гемоглобін
E. Довготривала гіпертензія
67.

67. After living abroad in Africa for a number of years, a 70-year-old man is seen by his physician because of
complaints of fatigue, weight loss, and anorexia. On physical examination, the physician notices a building
mass emerging from under the left costal margin, extending diagonally toward the right lower quadrant.
The patient also has several nontender cervical lymph nodes measuring up to 4 cm in size. Results of a
lymph node biopsy are shown.

What is the most likely diagnosis?


A. Diffuse large B-cell lymphoma
B. Follicular lymphoma
C. Infectious mononucleosis
D. Richter syndrome
E. *Small lymphocytic lymphoma

67. Після проживання за кордоном в Африці ряд років, чоловіка бачить його лікар тому що пацієнт
має скарги на втому, втрату ваги, і анорексії. На медичному огляді, лікар звертає увагу на набряклу
масову появу з-під лівої ребрової боковини, витягаючи виточки по діагоналі у напрямі правильного
нижчого квадранта. Пацієнт також має декілька твердих потиличних лімфатичних вузлів,
реабілітовуючих 4 см за розміром. Результати біопсії лімфатичного вузла показані.

Визначте ймовірний діагноз:

A. Дифузна величезна В-кліткова лімфома


B. Фолікулярна лімфома
C. Інфекційний мононуклеоз
D. Синдром Рітчера
E. *Маленька лімфатична лімфома
68.

68.A 63-year-old woman comes for evolution because of a 6-month history of increasing fatigue, weakness,
and anorexia. She states that she enjoyed playing tennis with her friends on weekends, but has not played
recently due to her inability to get through a set before ``tiring out``. On physical examination, sighificant
conjunctival pallor is noted. She also has petechiae scattered all over her body. Results of laboratory
studies and blood smear are as follows:

Hemoglobin: 7.3 g/dL


Platelet count: 90,000/mm^3
Leukocytes: 64,000/mm^3
Differential Neutrophils: 70%
Eosinophils: 5%
Lymphocytes: 20%
Bands 3%
Promyelocytes 2% and 1% myelocytes. No blasts are seen.

Which of the following is the most appropriate treatment?


A. Daunorubacan and cytarabine
B. Fludarabine and rituximab
C. *Imatinib
D. Leucovorin
E. Observetion

68. Жінка 63-х років заходить за оцінкою через 6-місячну історію збільшення втоми, слабкості, і
анорексії. Вона заявляє, що вона насолодилася грою в теніс з її друзями на вихідних днях, але не грав
нещодавно до її нездатності, щоб пройти через набір перед '' стомленням ''. На медичному огляді,
істотна кон'юнктивальна блідість відмічена. На її тілі також розсіяні петіхії скрізь. Результати
лабораторних вивчень і кров'яного мазка є як зазначено нижче:

Гемоглобін: 7.1
Тромбоцити: 90,000/mm3
Лейкоцити: 64,000/mm3
Диференціальні: нейтрофіли-70%, лімфоцити -20%, еозинофіли -5%, групи-3%, про мієлоцити-2%,
мієлоцити-1%. Бластних клітин не знайдено.

Який з наступного - саме відповідне лікування?


A. Дауноробіцин та цитарабін
B. Флударабін та рітуксімаб
C. *Іматініб
D. Лейковорин
E. Спостереження
69.

69. A 52-year-old woman comes to the doctor because of fatigue and pain in her hands. She was diagnosed
with diabetes a few years ago but has otherwise been healthy. On physical examination, the physician
notes hyperpigmentation on the skin over the patient`s hands and forearms as well as a palpable liver. The
Patient was adopted and does not know her family medical history. Initial laboratory results are as follows:
Na+: 140 mEq/L
K+: 3.9 mEq/L
BUN: 19 mq/dL
Cr: 0.65 mq/dL
AST: 100U/L
ALT: 112 U/L
Gluose: 145 mg/dL
On further analysis, which of the following laboratory abnormalities is most likely to be seen in this patient?
A. Decreased serum ceruloplasmin
B. Decreased serum ferritin
C. *Decreased total iron-binding capacity
D. Decreased transferrin saturation
E. Increased total iron-binding capacity
69.Жінка 52 років прийшла до лікаря через втому та біль в її руках . В неї був діагностовано діабет
через декілька років, але в іншому пацієнтка здорова. На медичному огляді, лікар відмічає
гіперпігментацію на шкірі руки пацієнта і заздалегідь відзначають також як пальпується печінка.
Пацієнт був прийнятий і не знає її сімейний анамнез. Початкові лабораторні результати є як зазначено
нижче.
Na+: 140 mEq/L
K+: 3.9 mEq/L
BUN: 19 mq/dL
Cr: 0.65 mq/dL
AСТ: 100 U/L
AЛT: 112 U/L
Глюкоза: 145 mg/dL
На подальшому аналізі, який з наступних лабораторних ненормальностей ймовірно бачитися у цього
пацієнта?

A. Зменшився церум плазмід


B. Зменшився церум феритин
C. *Зменшився повна iron-binding місткість
D. Зменшилась сутурація трансферину
E. Збільшилась повна iron-binding місткість
70.
70. Африко-американський хлопчик 10 років прийшов до кімнати невідкладної допомоги через
випадковий, інтенсивний грудний біль, хрипи та кашель. Його мати стверджує що він грав з його
старшим братом і це передує його з’явившимся симптомам. Він має подібний епізод одного разу
перед тим, як з'явився цей епізод. Пацієнт не має ніяких інших медичних умов і сімейна історія
нечудова. Його життєво важливі показники включають температуру 38.7 С, кровяний тиск 120/85 mm
Hg, пульс 98/хв., дихальні показники 22/хв.. Ніякі дзюрчання не виявлені і легеня аускультативно
чиста з обох боків. Живот пацієнта з'являється м'яко і неніжний. Край селезінки був
прапальпований. Результати рентгенограми живота показані в зображенні .

Лейкоцити: 5.1 *10^9/л


Еритроцити:4.5*10^12/л
Гемоглобін: 10.9 гр/л
Гемотакрит: 39%
Середній об’єм еритроцита: 83fL
Середня концентрація гемоглобіна: 35,9%
Ширина розподілу еритроцитів: 14.7
Тромбоцити: 250*10^9/л
Пацієнта було негайно гідратовано внутришньовеним розчином та було ініційовано управління
біллю.

Який з наведених нижче механізмів найбільш краще пояснює стан пацієнта?


A. *Ненормальна закупорка маленьких судин еритроцитами
B. Апластичний криз парвовірусов В 19
C. Закупорка коронарних артерій
D. Викид lgE-медіаторів гистаміну
E. Чутливість інкапсульованих организмів приводить до дихальної інфекції
71.

71. A 60-year-old woman with recent history of Roux en-y gastric bypass comes for evaluation because of
fatigue and shortness of breath. She complains of generalized weakness and numbness in her fingertips and
toes. She also has noticed increased bloating, diarrhea, and abdominal pain. She denies any alcohol

use and, other than obesity, has no significant medical history. Conjunctive are pale, there is loss of position
sense on the second toe bilaterally, and the patient's gait appears unsteady. The physician orders additional
tests including a CBC which shows a Hib of 9.3 and an MCV of 110. A hydrogen breath test is positive.

Which of the following laboratory results supports the patient's probable diagnosis?

A. Decreased homocysteine level


B. Decreased level of lactate dehydrogenase
C. Elevated WBC count
D. *Increased methylmalonic acid
E. Positive anti-intrinsic factor antibody assay.
F. Schilling test within normal limits

71. 60-річна жінка з недавньою історією шунтування шлунка по Ру-Y звернулася за оцінкою стану
через почуття втоми та задишку. Вона скаржиться на загальну слабкість і оніміння пальців рук та ніг.
Також вона помітила підвищений метеоризм, діарею і біль в животі. Жінка заперечує будь-яке
вживання алкоголю і не має значного анамнезу хвороби, окрім ожиріння. Спостерігаються бліді
кон’юктиви, двостороння втрата чутливості на другому пальці ноги і нестійка хода пацієнта. Лікар
призначив додаткові аналізи в тому числі повний аналіз крові, який показав вміст гемоглобіна – 9,3 і
MCV (середній об’єм еритроциту) – 110. Дихальний водневий тест позитивний.

Які з наступних лабораторних результатів підтверджують ймовірний діагноз пацієнта?

А. Зниження рівня гомоцистеїну

B. Зниження рівня лактатдегідрогенази

С. Підвищення числа лейкоцитів


D. *Збільшення метилмалонової кислоти

Е. Позитивна проба на антитіла до внутрішнього фактору Кастла

F. Проба Шилінга в межах норми


72.

72. A 48-year-old woman presents to her physician with a 2-month history of dull left upper quadrant pain
and chronic fatigue. Laboratory test results are as follows:

WBS count: 38 ×109 /L

Neutrophils: 95%
Basophils: 2%

RBC count: 1,8×1012/L

Platelet count: 600,000/mm 3

A bone marrow biopsy is then performed, and the aspirate smear is shown. Genetic testing confirms the
diagnosis, and the patient is treated with a drug that inhibits the oncogene product in this disease.

Which of the following signaling pathways does this drug inhibit?

A. G protein coupled receptor signaling

B. NF-kB signaling

C. Steroid receptor signaling

D. TGF- β receptor signaling

E. *Tyrosine kinase signaling

72. 48-річна жінка звернулася до свого лікаря з 2-місячною історією тупого болю в лівому квадранті і
хронічною втомою. Результати лабораторних випробувань наступні:

Лейкоцити: : 38 ×109

Нейтрофіли: 95%

Базофіли: 2%

Еритроцити: 1,8×1012

Тромбоцити: 600,000/м м 3

Після цього була виконана біопсія кісткового мозку і показаний мазок з пунктату. Генетичне
тестування підтверджує діагноз, пацієнт лікується препаратом, який блокує продукт онкогена в цьому
захворюванні.
Який з наступних сигнальних шляхів інгібує даний препарат?

А. Сигнальні рецептори, зв'язані з G-білком

В. Сигнальний шлях NF-kB (ядерний фактор «каппа-бі»)

С. Сигнальні стероїдні рецептори

D. Сигнальні рецептори TGF- β (трансформуючого ростового фактору бета)

E. *Тирозинкіназа
73.
73. A 32-year-old woman comes to the physician complaining of new bruises on her legs, but doesn't recall
any accidents or injuries which could have produced them. She says that she is also bothered by a rash on
both arms, and she's worried that she will not be able to wear a sleeveless dress at her sister's wedding
next weekend

On physical examination, the physician notes a diffuse rash composed of small red dots. The rash does not
blanch when he presses down on it. The liver and spleen are not palpable. Strength, sensation, and reflexes
are normal bilaterally. The rash is shown.

The physician decides to order laboratory tests, which reveal:

WBC: 8000/mm 3

Hemoglobin: 13 g/dl.

Hematocrit: 39%

Platelets: 20,000/ mm 3

INR: 1.0

Cro. 7

BUN: 12

Following a course of therapy, the patient`s symptoms resolve in 3 weeks.

Which of the following was the most likely diagnosis?

A. Disseminated intravascular coagulation

B. Hemophilia A

C. *Immune thrombocytopenic purpura

D. Thrombotic thrombocytopenic purpura

E. von Willebrand disease

73. 32-річна жінка звернулася до лікаря зі скаргою на нові синці на ногах, але не пам'ятає ніяких
нещасних випадків або травм, які могли б їх викликати. Пацієнтку також турбує висип на обох руках, і
вона засмучується, що не зможе носити сукню без рукавів на весіллі своєї сестри у наступні вихідні.
При фізичному огляді, лікар відзначає дифузний висип, який складається з маленьких червоних точок
і не блідне при натисканні на нього. Печінка і селезінка не пальпуються. Сила, чутливість і рефлекси
нормальні з обох сторін. Фото висипу показано нижче.

Лікар замовив лабораторні аналізи, які виявили:

Лейкоцити: 8000 м м 3

Гемоглобін: 13 г/дл

Гематокріт: 39%

Тромбоцити: 20,000 м м 3

МНО: 1.0

Креатинін: 0.7

Сечовина: 12

Після курсу терапії симптоми у пацієнтки зникли через 3 тижні.

Який з наступних діагнозів був найбільш імовірним?

А. Дисеміноване внутрішньосудинне згортання

В. Гемофілія А

С. *Імунна тромбоцитопенічна пурпура

D. Тромботична тромбоцитопенічна пурпура

E. Хвороба фон Віллебранда


74.

74. A 29-year-old man presents to his physician with complaints of spontaneous bleeding in his gums and
blood in his urine. He also has fatigue and a low-grade fever. Physical exam notes purpura over the
extremities with a palpable spleen and enlarged liver. Laboratory values reveal:

Hemoglobin: 7g/dl.

Hematocrit: 21%

Platelet count: 25,000/mm 3


Leukocytes: 17,300/mm 3

Neutrophils: 64%

Eosinophils: 2%

Lymphocytes: 26%

Monocytes: 6%

Fibrinogen: 100 mg/dl (normal 150-400 mg/dL)

PT: 22 sec

PTT: 50 sec

What is the pathologist most likely to observe on this patient's peripheral blood smear?

A. Increased metamyelocytes

B. Lymphoblasts

C. Lymphocytes with smudge cells

D. *Myeloblasts with Auer rods

E. Normal peripheral blood smear

74. 29-річний чоловік звернувся до свого лікаря зі скаргами на спонтанну кровотечу ясен і кров у сечі.
Також він скаржиться втому і субфебрильну лихоманку. При огляді булла виявлена пурпура на
кунцівках. Пальпується селезінка і збільшена печінка. Значення лабораторії показують:

Гемоглобін: 7 г/дЛ

Гематокріт: 21%

Кількість тромбоцитів: 25,000/м м 3

Лейкоцити: 17,300/м м 3

Нейтрофіли: 64%

Еозинофіли: 2%

Лімфоцити: 26%

Моноцити: 6%

Фібриноген: 100 мг/дл (норма 150-400 мг/дл)


Протромбіновий час: 22 сек

PTT: 50 сек

Що патологоанатом швидше за все помітить на мазку периферичної крові цього пацієнта?

А. Збільшення метаміелоцитів

Б. Лімфобласти

C. Лімфоцити з тільцями Боткіна-Гумпрехта

Д. *Мієлобласти з тільцями Ауера

Е. Нормальний мазок периферичної крові


75.

75. An 18-year-old woman presents with a history of easy bruising. She denies any known comorbidity.
Coagulation studies are notable for a prolonged bleeding time and prolonged partial thromboplastin time
while a cofactor assay failed to elicit increased platelet aggregation. Results of a complete blood count
show the following:

Hemoglobin: 11.0 g/dL

Hematocrit: 33%

Leukocyte count: 5400/mm3

Platelet count: 200,000/mm3


The physician suspects a bleeding disorder.

Which of the following clinical findings is most consistent with the likely diagnosis?

A. Diffuse petechiae and purpura

B. *Excessive bleeding after extraction of an impacted molar

C. Hepatosplenomegaly

D. Knee pain and swelling after light trauma against a table

E. Schistocytes on peripheral blood smear

75.18-річна жінка звернулася до лікаря з легкими синцями. Вона заперечує всі відомі супутні
захворювання. При дослідженні коагуляції була виявлена тривала кровотеча і збільшення часткового
тромбопластинового часу, разом з тим аналіз кофактору не отримав збільшення агрегації
тромбоцитів. Результати повного аналізу крові показують наступне:

Гемоглобін: 11.0 г/дл

Гематокріт: 33%

Лейкоцити: 5400/ мм 3

Тромбоцити: 200,000/ мм 3

Лікар підозрює порушення згортаємості крові.

Які з наступних клінічних результатів найбільш узгоджуються з імовірним діагнозом?

А. Дифузні петехії і пурпура

В. *Рясна кровотеча після вилучення пошкодженого моляру.

С. Гепатоспленомегалія

D. Біль в коліні і набряки після легкої травми

E. Шистоциты на мазку периферичної крові


76.
76. A.14-year-old boy is brought to the emergency department after his parents notice a yellow
discoloration of his eyes. They say that their son recently had a case of mild diarrhea, which the family
treated with a leftover antibiotic. Laboratory studies show:

Hemoglobin: 11 g/dL

Hematocrit: 32%

Platelet count: 250,000/mm3

WBC. count: 7000/mm3

Reticulocyte count: 2.8%

Indirect bilirubin: 3.6 mg/dL

Results of a Coombs test are negative. Peripheral blood smear with staining is shown below.

Which of the following is the most likely cause of this patient's anemia?

A. Aplastic anemia

B. Autoimmune hemolytic anemia

C. *Glucose-6-phosphate dehydrogenase deficiency

D. Hereditary spherocytosis

E. Sickle cell disease

76.14-річний хлопчик доставлений у відділення невідкладної допомоги після того, як його батьки
помітили пожовтіння його очей. Вони кажуть, що у їхнього сина нещодавно був випадок легкої діареї,
яку сім'я лікувала залишками антибіотика. Лабораторні дослідження показують:

Гемоглобін: 11 г/дЛ

Гематокрит: 32%

Кількість тромбоцитів: 250,000/ мм 3

Кількість лейкоцитів: 7000/ мм 3

Кількість ретикулоцитів: 2.8%


Непрямий білірубін: 3,6 мг/дл

Результати тесту Кумбса негативні. Мазок периферичної крові з фарбуванням показаний нижче.

Що з перерахованого нижче є найбільш ймовірною причиною анемії цього пацієнта?

А. Апластична анемія

B. Аутоімунна гемолітична анемія

C. *Дефіцит глюкозо-6-фосфатдегідрогенази

D. Спадковий сфероцитоз

Е. Серповидно-клітинна анемія
77.

77. An 18-year-old woman is brought to the urgent care clinic because of progressive fatigue and dyspnea
for the past 2 weeks. She has a history of sickle cell disease diagnosed in early childhood, and she has had
multiple hospitalizations every year for pain. She has received three blood transfusions in the past; the last
one was years are, Menses occur monthly with heavy flow for 5 days. She received a 7 day course of
amoxicillin/clavulanate for a sinus infection 3 weeks ago. Her only other medication is occasional ibuprofen.
She does not smoke but drinks two beers per night on some weekends. Temperature is 36.1"C (97"F),
blood pressure is 100/50 mm Hg, pulse is 110/min; pulse oximetry on room air shows an oxygen saturation
of 98%... Physical examination shows pale conjunctivae without scleral icterus; there are no abnormalities
on cardiac, abdominal, or neurological examinations. Laboratory studies show;
Hemoglobin: 5.7 g/dl.

WBCs: 5000/mm3

Platelets: 200/mm3

Mean corpuscular volume: 118 fL

Total bilirubin: 1.3 mg/dl.

Methylmalonic acid: 10 umol/L (N - 0-40 mol/L)

Electrolyte and creatinine concentrations are within reference ranges. X ray of the chest shows no
significant findings. The patient is sent to the emergency department for a blood transfusion.

Which of the following is the most likely cause of this patient's blood test results?

A. Adverse effect of medication

B. Alcohol intake

C. Anemia of chronic disease

D. *Folate deficiency

E. Orotic aciduria

F. Vitamin B12 deficiency

77. 18-річна жінка доставлена в клініку невідкладної допомоги через прогресуючу втому і задишку
протягом останніх 2 тижнів. У неї в анамнезі серповидноклітинна хвороба, діагностована в ранньому
дитинстві, також було кілька госпіталізацій щороку через біль. Ій було виконано три переливання
крові в минулому; останній був рік тому, менструації відбуваються щомісяця з важким перебігом
протягом 5 днів. 3 тижні тому вона отримала 7-денний курс амоксициліну/клавуланату для синусової
інфекції. Її єдині ліки-Ібупрофен. Вона не курить, але п'є по два пива на ніч у вихідні. Температура 36,1
"С " (97 Ф), кров'яний тиск 100/50 мм рт ст., пульс 110/хв, пульсоксиметрія повітря в приміщенні
показує насичення киснем 98%.

Фізичний огляд показує блідість кон'юнктиви без жовтизни склер. При обстеженні не виявлено
кардіальних, абдомінальних та неврологічних порушень. Лабораторні дослідження показують:

Гемоглобін: 5.7 г/дл

Лейкоцити: 5000/ м м 3

Тромбоцити: 200 / м м 3

Середній об'єм еритроциту: 118 фЛ

Загальний білірубін: 1.3 мг/дл.


Метилмалонова кислота: 10 моль / л (норма 0-40 моль / л)

Концентрації електроліту і креатиніну знаходяться в межах еталонних значень. Рентгенівський знімок


грудної клітини не виявив значущих результатів. Пацієнтку направлено у відділення невідкладної
допомоги для переливання крові.

Що з перерахованого є найбільш ймовірною причиною результатів аналізу крові цього пацієнта?

А. Негативний вплив ліків

В. Споживання алкоголю

С. Анемія при хронічній хворобі

D. *Дефіцит фолатів

Е. Оротова ацидурия

F. Дефіцит вітаміну B12


78.

78.A 79-year-old woman is brought by ambulance from her nursing home to the emergency department.
The EMTs were unable to obtain any history; her vital signs en route to the hospital are as follows:

Temperature: 39.6°C (103°F)

Pulse: 115/min

Blood pressure: 86/49 mm Hg

Respiratory rate: 22/min

O2 saturation: 100% on room air


On evaluation, she has an altered mental status and is unable to provide any history. The physician
completes a full body physical examination, which reveals diffuse petechiae and purpura. During the
examination the patient begins to bleed from her nose and requires intranasal packing to stop the bleeding.

Laboratory studies show:

WBC: 16.2× 109 / L

Platelets: 15,000/mm 3

Hemoglobin: 11.g/dL

Hematocrit: 33%

Creatinine: 1.2 mg/dL

INR: 1.7

PT: 20 sec

PTT: 45 sec

Urine nitrites: positive

She is given a fluid bolus, started on empiric antibiotics, and transferred to the intensive care unit.

Which of the following pathophysiological mechanisms is most likely the cause of this patient's condition?

A. Absence of protease responsible for cleaving von Willebrand factor

B. Antibodies directed at RBC membrane antigens

C. Antibodies directed at platelet membrane antigens

D. Toxin-producing bacterial infection

E. *Widespread activation of the coagulation cascad

78. 79-річна жінка доставлена на машині швидкої з будинку престарілих у відділення невідкладної
допомоги. Лікарі швидкої не змогли отримати анамнезу, по дорозі в лікарню її життєво важливі
симптоми це:

Температура: 39.6°C (103°Ф)

Пульс: 115 / мін

Артеріальний тиск: 86/49 мм рт. ст.

Частота дихання: 22/хв

Сатурація О2 : 100% в повітрі кімнати


На обстеженні вона має зміну психічного статусу і не може надати ніякого анамнезу. При повному
фізичному обстеженні лікарем були виявлені дифузні петехії і пурпура. Під час маніпуляцій у
пацієнтки почалася кровотеча з носа і для того, щоб ії зупинити потрібна інтранозальна тампонада.

Лабораторні дослідженні показали:

Лейкоцити: 16.2× 109 / л

Тромбоцити: 15,000/ мм 3

Гемоглобін: 11 г/дЛ

Гематокрит: 33%

Креатинін: 1.2 мг/дЛ

МНО: 1.7

Тромбопластичний час: 20 сек

Частковий тромбопластичний час: 45 секунд

Нітрити в сечі: позитивний

Їй ввели внутрішньовенні ліки, призначили емпіричні антибіотики и перевели у реанімацію.

Який з наступних патофізіологічних механізмів є найбільш ймовірною причиною цього


захворювання?

A. Відсутність протеази, відповідальної за розщеплення фактора Віллебранда

B. Антитіла, проти антигенів еритроцитів

C. Антитіла проти антигенів тромбоцитів

D. Токсин-продукуючі бактеріальні інфекції

E. *Поширена активація коагуляційного каскаду


79.
79.

A 57-year-old man presents to his internist because of abnormal bleeding after a routine dental cleaning.
Over the past several months, he has experienced symptoms of headache, dizziness, fatigue, decreased
vision, and occasional nosebleeds. He has also begun noticing "lumps" in his armpits and groin region.

The spleen tip is palpable on physical examination. Laboratory test results are as follows:

Hgb: 11 g/dL

BUN: 25 mg/dL

Calcium: 9.1

Total protein: 15 g/dL

Immunofixation shows high levels of IgM. The findings from his bone marrow biopsy specimen are shown in
the image.

Which of the following is the most likely explanation for the patient's symptoms?

A. Heavy-chain disease

B. Monoclonal gammopathy of undetermined significance

C. Multiple myeloma

D. Primary amyloidosis

E. *Waldenstrom macroglobulinemia

79. 57-річний чоловік звернувся до свого терапевта через аномальну кровотечу після планової чистки
зубів. За останні кілька місяців, він відмітив симптоми головного болю, запаморочення, втоми,
погіршення зору і випадкові носові кровотечі. Також в нього з'явилися "грудки" в області пахв і паху.
При фізичному огляді пальпується кінчик селезінки. Результати лабораторних досліджень наступні:

Гемоглобін: 11г/дЛ

Сечовина в крові: 25 мг/дЛ

Кальцій: 9,1
Загальний білок: 15 г/дЛ

Іммунофіксація показала високі рівні IgM, результати біопсії кісткового мозку представлені на
малюнку.

Яке з наступних пояснень є найбільш імовірним для симптомів пацієнта?

А. Хвороба важких ланцюгів

В. Моноклональна гаммопатія невизначеного значення

С. Множинна мієлома

D. Первинний амілоїдоз

Е. *Макроглобулінемія Вальденстрема
80.

80. A 60-year-old man presents with complaints of fatigue and a sporadic history of unexplainable bruising.
He reports no recent weight loss and no recent trauma to explain the bruising but does say that he has
been very tired lately. He has also been experiencing lower extremity bone pain and joint tenderness. He
reports that he drinks three beers each night and has smoked about one pack of cigarettes per day for 20
years. Physical examination reveals a palpable spleen, as well as red spots on the arms and thighs,
approximately I cm in diameter, which do not blanch on application of pressure. Laboratory testing reveals
the following:

Hemoglobin: 6g/dL

Hematocrit: 18%

Platelet count: 70,000/mm3


Leukocytes: 1300/mm3

Neutrophils: 58%

Eosinophils: 2%

Lymphocytes: 34%

Monocytes: 6%

Fibrinogen: 150 mg/dL

INR: 1.1

The patient's physician attempts a bone marrow aspiration but is unable to obtain a sample.

Which of the following would most likely be seen on a peripheral blood smear?

A. Acanthocytes

B. Macro-ovalocytes

C. Schistocytes

D. Spherocytes

E. Target cells

F. *Teardrop cells

80.60-річний чоловік скаржиться на втому і одиничне захворювання з незрозумілими синцями.


Пацієнт повідомляє, що недавньої втрати ваги та недавньої травми не було, щоб пояснити синці, але
каже, він дуже стомлюється останнім часом. Чоловік також відчуває біль в кістках нижніх кінцівок і
хворобливість суглобів. Він розповів, що п'є три пива щоночі і викурює близько однієї пачки сигарет в
день протягом 20 років. При фізичному огляді пальпується селезінка, а також визначено червоні
плями на руках і стегнах, приблизно 1 см в діаметрі, які не бліднуть при натисканні. Лабораторні
досліження показують наступне:

Гемоглобін: 6г/дЛ

Гематокрит: 18%

Кількість тромбоцитів: 70,000/ мм 3

Лейкоцити: 1300 / мм 3

Нейтрофіли: 58%

Еозинофіли: 2%

Лімфоцити: 34%

Моноцити: 6%

Фібриноген: 150 мг/дл

МНО: 1.1
Лікар цього пацієнта здійснив пункцію кісткового мозку, але не отримав зразок.

Що з перерахованого, швидше за все, можна побачити на мазку периферичної крові?

A. Акантоцити

B. Макро-овалоцити

C. Шистоцити

D. Сфероцити

E. Клітини-мішені

F. *Каплевидні клітини
81.

81.An 8-month girl is brought to the pediatrician’s office because her parents are worried a decrease in the child’s
activity. The mother notes the child is feeding well and produces an adequate number of wet diapers daily. The mother
is able to adequately breastfeeds the child every 3 hours, with no need to supplement with formula. The child has not
been introduced to baby food yet but states she plans on starting to introduce baby food soon. On physical examination ,
the child appears very lethargic . Results of a complete blood count show a hemoglobin level of 8.4 gm/dL

Which of the following findings would most likely be present in association with this patient’s anemia?

A. *Decreased mean corpuscular volume


B. Decreased total iron-binding capacity
C. Hypocellular marrow
D. Increased ferritin level
E. Increased reticulocyte count
F. Macrocytosis

81.8-місячну дівчину привозять до кабінету педіатра, тому що її батьки стурбовані зниженням активності
дитини. Мама відзначає, що дитина добре годується і виробляє достатню кількість мокрих пелюшок щодня.
Мати здатна адекватно годувати дитину кожні 3 години, без необхідності доповнювати її штучними сумішами.
Дитина ще не вживала дитяче харчування, але мати стверджує, що найближчим часом вона планує вводити
дитяче харчування. При медичному огляді дитина виявляється дуже млявою. Результати загального аналізу
крові показують рівень гемоглобіну 8,4 г / л

Які з наведених відповідей, ймовірно, будуть присутні у зв'язку з анемією цього пацієнта?

A. *Знижений середній корпускулярний об'єм

B. Зниження загальної здатності до зв'язування заліза

С. Гіпоцеллюлярний кістковий мозок

D. Збільшення рівня феритину

E. Підвищена кількість ретикулоцитів


F. Макроцитоз
82.

82. A 10-month-old infant is brought to the doctor because he has had a cough and runny nose for the past 3 days. On
physical examination , the infant’s eyes and skin appears yellow, and the spleen tip is palpated in the left lower quadrant
of the abdomen . Laboratory studies are ordered and a peripheral smear is obtained and show.

Which of the following characteristics is most likely to be seen on further hematologic analysis?
A. Hypersegmented polymorphonuclear cells
B. *Increased mean corpuscular hemoglobin concentration
C. Increased total iron-bindind capacity
D. Positive direct Coombs test
E. Rouleaux formation of RBCs

82. 10-місячну дитину доставляють до лікаря, тому що він має кашель і нежить протягом останніх 3 днів. При
медичному огляді очі і шкіра мають жовте забарвлення і пальпується кінчик селезінки в лівому нижньому
квадранті живота.У якості лабораторного дослідження було взято переферичний мазок.

Які з наведених нижче характеристик найбільш імовірно можна побачити на подальшому гематологічному
аналізі?

А. Гіперсегментовані поліморфноядерні клітини

B. *Збільшення середньої концентрації корпускулярного гемоглобіну

C. Підвищена загальна зв’язувальна здатність заліза

D. Позитивний тест Кумбса

E. Згорнуті формування еритроцитів


83.

83. A 70-years-old man with yellow-white «bumps» on his ears and Achilles tendon presents to the clinic .
He has a long history of pain in his ankles , toes, and fingers and reports that he has had extremely painful
flare-ups every few month, which were following by periods when he was free of symptoms. However, in
recent years, he has had near-constant joint pain.

On examination, the patient’s lesions are palpated but not tended to the touch. The ear lesions do not
transilluminate. Moreover, there is significant swelling of the joints, especially the toes and knees.

What is the mechanism of action of the medication that can reduce the risk of developing such lesions and
be used for long-term management of this patient’s condition?

A. Binds tubulin
B. Block formations of prostaglandins and thromboxane from arachidonic acid
C. Inhibits release of phospholipase A2
D. *Inhibits xanthine oxidase
E. Selectively inhibits cyclooxygenase-2

83.70-річний чоловік з жовто-білими «шишками» на вухах і ахілловому сухожиллі звертається до


клініки . Він має біль вже дуже довгий час у щиколотках, великих пальцях ніг і пальцях рук і каже, що
він мав надзвичайно хворобливі спалахи кожні кілька місяців, які слідували за періодами, коли він не
мав симптомів. Проте в останні роки він мав майже постійні болі в суглобах.

При огляді пальпувалися ділянка що турбували пацієнта і вони були тверді на дотик. Ушкодження
вуха не просвічують. Крім того, спостерігається значна набряклість суглобів, особливо пальців і колін.

Який механізм дії препарату може знизити ризик розвитку таких уражень і використовувати його для
тривалого лікування стану пацієнта?

A. Зв’язуючі тубулін

Б. Блочні утворення простагландинів і тромбоксану з арахідонової кислоти


C. Засоби що пригнічують вивільнення фосфоліпази А2

D. *Засоби що пригнічують ксантиноксидазу

E. Селективні інгібітори циклооксигенази-2


84.

84. A 26-year-old African-American man presents to a travel clinic for a consultation. He explains that he is
going to Uganda as a part of medical mission to provide free antiviral therapy for individuals infected with
HIV. He is requesting information regarding vaccines and prophylactic treatment for infectious diseases he
may be exposed to while there. The physician administers typhoid vaccine, yellow fever vaccine, and gives
the patient a prescription for antimalarial pills, instructed him to begin the medication several days before
he arrives in Africa. Three days after beginning the medication , the patient begins to feel excessively
fatigue and noticed that his urine is much darker than usual.

Which of the following represents the pathophysiologic basis for the patient’s current condition?

A. Deficient synthesis of a-globin chain


B. Increased sensitivity of RBCs to lytic activity of complement
C. *Intracellular accumulation of peroxides
D. Intrincis RBC membrane defect
E. Point mutation in the hemoglobin gene

84.26-річний афро-американець звертається у туристичну клініку на консультацію. Він пояснює, що


він збирається в Уганду в рамках медичної місії, щоб забезпечити безкоштовну противірусну терапію
для осіб, інфікованих ВІЛ. Він запитує інформацію про вакцини та профілактичне лікування
інфекційних захворювань, яким може бути підданий в той час. Лікар виписує тифозну вакцину,
вакцину проти жовтої лихоманки і дає пацієнту рецепт протималярійних таблеток, доручив йому
розпочати прийом ліків за кілька днів до прибуття в Африку. Через три дні після початку лікування
пацієнт починає відчувати надмірну втому і помічає, що його сеча набагато темніше, ніж зазвичай.

Яке з наведеного нижче являє собою патофізіологічною основою для чинного стану пацієнта?

A. Дефіцитний синтез а-глобінового ланцюга

Б. Підвищена чутливість еритроцитів до літичної активності комплементу


C. *Внутрішньоклітинне накопичення пероксидів

D. Дефект внутрішньої мембрани еритроцитів

E. Точкова мутація гена гемоглобіну


85.

85. A73-year-old woman with a history of metastatic ovarian cancer presents to the emergency department
with an incessant nosebleed that has lasted for the past 2 hours. She has not received chemotherapy for
over a month because she was unable to tolerate the side effects. After inserting nasal packing and gaining
control of the bleeding the physician performs a complete physical examination. He notes f heart rate of
99/min; respiratory rate of 18/min and nodular, tender, palpable cards in her inner thighs , bilaterally.

The physician initiates intravenous fluid resuscitation and notices that blood continues to ooze around the
venipuncture side. Hematologic test reveal the following:

WBC 11x 103 /mL

Hemoglobin 8.9 g/dL


Hematocrit 29%

Platelets 59x 103/mL

INR 1.4

Which of the following additional laboratory findings is most likely to be seen in this patient?

A. Decreased fibrin split products


B. Decreased partial thromboplastin time
C. *Elevated D-dimer
D. Increased fibrinogen
E. Normal bleeding time

85. 73-річна жінка з метастатичним ракам яєчників у анамнезі звертається до відділення невідкладної
допомоги з постійною кровотечею з носа, що триває протягом останні 2 години. Вона не отримувала
хіміотерапію більше місяця, тому що вона не могла перенусти побічні ефекти. Після встановлення
назальної тампонади і встановлення контролю над кровотечею, лікар виконує повний медичний
огляд. Він відзначає частоту серцевих скорочень 99 / хв; частота дихання 18 / хв і вузловаті, ніжні, на
дотик утворення на внутрішніх ділянках стегон двосторонньо.

Лікар ініціює внутрішньовенну реанімацію рідини і помічає, що кров продовжує просочуватися


навколомісця ін’єкції. Гематологічне дослідження виявило наступне:

Білі кров’яні тільця 11х103 / мл

Гемоглобін 8,9 г / л

Гематокрит 29%

Тромбоцити 59х103 / л

МНВ 1.4

Які з наступних показників, найвірогідніше, можна побачити у цього пацієнта в лабораторних


дослідженнях?

A. Зниження продуктів поділу фібрину

B. Зменшення часткового тромбопластинового часу

C. *Підвищений D-димер

D. Збільшення фібриногену

E. Нормальний час кровотечі

You might also like